Location via proxy:   [ UP ]  
[Report a bug]   [Manage cookies]                

Passable 350 Q&a

Download as pdf or txt
Download as pdf or txt
You are on page 1of 76

PLAB 1

SINGLE BEST ANSWER

Al-Khair
1. A homeless alcoholic with a chronic productive cough, presents with haemoptysis. The chest
x-ray shows a cavitating lesion
What is the single most likely causative organism?

a. Chlamydia psittaci
b. Mycobacterium tuberculosis
c. Staphylococcus aureus
d. Haemophilus influenzae
e. Streptococcus pneumoniae

2. A 50 year old man became suddenly breathless whilst eating. He has marked stridor and is
choking and drooling.
What is the single most next appropriate step in management?

a. IV aminophylline
b. Plueral aspiration
c. Heimlich maneuver
d. Chest drain
e. IV furosemide

3. A 60 year old man presents with acute breathlessness and a cough productive of frothy, pink
sputum. He cannot lie flat. On examination he has crackles in both mid-zones and a few
scattered wheezes.
What is the single most next appropriate step in management?

a. Chest drain
b. IV frusemide
c. Rapid infusion of saline
d. Forced alkaline diuresis
e. Intramuscular adrenaline

4. A 20 year old woman is too breathless to speak. Her pulse is 120/min respiratory rate 30/min
and PEFR is 100L/min. examination reveals a very quiet chest and CXR is normal.
What is the single most next appropriate step in management?

a. Nebulisd salbutamol
b. IV aminophylline
c. Plueural aspiration
d. Heimlich maneuver
e. Chest drain

5. A chronic smoker with COPD develops a fever. He reports bringing up green phlegm on
coughing.
What is the single most next appropriate step in management?

a. Staphylococcus aureus
b. Haemophilus influenzae
c. Streptococcus pneumoniae
d. Chlamydia trachomatis
e. Mycoplasma pneumoniae

6. A young male homosexual with Kaposi’s sarcoma complains of increasing breathlessness and
a dry
What is the single most likely causative organism?

a. Chlamydia trachomatis
b. Mycoplasma pneumoniae
c. Pneumocystis carinii
d. Legionella pneumoniae
e. Klebsiella pneumoniae

7. A 25 year old man has a 3 day history of shivering, general malaise and productive cough. The
x-ray shows right lower lobe consolidation.
What is the single most likely causative organism?

a. Mycobacterium tuberculosis
b. Mycoplasma pneumoniae
c. Pneumocystis carinii
d. Staphylococcus aureus
e. Streptococcus pneumoniae

8. A 26 year old man presents with severe shortness of breath and dry cough which he has had
for 24hrs. He is very distressed. He has been on IV drug abuser. The x-ray shows perhilar fine
mottling.
What is the single most likely causative organism?

a. Mixed growth of organisms


b. Mycobacterium tuberculosis
c. Mycoplasma pneumoniae
d. Pneumocystis carinii
e. Staphylococcus aureus

9. A 35 year old previously healthy man returned from holiday five days ago. He smokes 10
cigarettes per day. He presents with mild confusion, a dry cough and mild pyrexia. His chest is
normal. The x- ray shows widespread upper zone shadowing.
What is the single most likely causative organism?

a. Legionella pneumophila
b. Mixed growth of organism
c. Mycobacterium tuberculosis
d. Mycoplasma pneumoniae
e. Pneumocystis carinii
10. A 20 year old previously healthy woman presents with general malaise, severe cough and
breathlessness, which has not improved with a 7 day course of amoxicillin. There is nothing
significant to find on examination. The x-ray shows patchy shadowing throughout the lung
fields. The blood film shows clumping of red cells with suggestion of cold agglutinins.
What is the single most likely causative organism?

a. Mixed growth of organisms


b. Mycobacterium tuberculosis
c. Mycoplasma pneumoniae
d. Pneumocystis carinii
e. Staphylococcus aureus

11. A 35 year old woman presents with 4 month history of cough, productive and recent
haemoptysis. She has lost 5kg in weight. The CXR shows right upper lobe consolidation.
What is the single most likely diagnosis?

a. Streptococcus pneumoniae
b. Influenza
c. Tuberculosis
d. Klebsiella pneumoniae
e. Viral

12. A previously well 18 year old has had influenza for the last 2 weeks. She is deteriorating and
has a swinging fever. She is coughing up copious purulent sputum. CXR shows cavitating
lesions.
What is the single most likely diagnosis?

a. Influenza
b. Tuberculosis
c. Klebsiella pneumoniae
d. Viral
e. Lung cancer

13. A 65 year old man currently undergoing chemotherapy of chronic leukaemia has felt unwell
with fever and unproductive cough for 2 weeks despite treatment with broad spectrum IV
antibiotics. The CXR shows an enlarging right sided mid zone consolidation.
What is the single most likely diagnosis?

a. Fungal
b. Streptococcus pneumoniae
c. Influenza
d. Tuberculosis
e. Klebsiella pneumoniae

14. A 27 year old male prostitute has felt generally unwell for 2 months with some weight loss.
Over the last 3 weeks he has noticed a dry cough with increasing breathlessness. Two courses
of antibiotics from the GP have produced no improvement. The CXR shows bilateral interstitial
infiltrates.
What is the single most likely diagnosis?

a. Streptococcus pneumoniae
b. Haemophilus influenzae
c. Tuberculosis
d. Klebsiella pneumoniae
e. Pneumocystis carinii

15. On return to university, a 20 year old student presented with the onset of fever, malaise and
a dry cough. The student health service gave him amoxicillin. After a week he felt no better
and his CXR showed patchy bilateral consolidation.
What is the single most likely diagnosis?

a. Klebsiella pneumoniae
b. Viral
c. Lung cancer
d. Legionella pneumoniae
e. Mycoplasma pneumoniae

16. A 64- year old man with COPD presents with pneumonia clinically he improves with antibiotics.
In the outpatient clinic 4 weeks later, the consolidation on his CXR has not resolved.
What is the single most likely diagnosis?

a. Haemophilus influenzae
b. Tuberculosis
c. Klebsiella pneumoniae
d. Viral
e. Lung cancer

17. A 27- year old male patient has just returned from holiday abroad presents with flu like illness,
headache, and high fever prior to this, he had complained of abdominal pain, vomiting,
diarrhoea associated with blood per rectum.
What is the single most likely diagnosis?

a. Klebsiella pneumoniae
b. Viral
c. Lung cancer
d. Legionella pneumoniae
e. Mycoplasma pneumoniae

18. A 30 year old obese man complains of retrosternal chest pain worse at night and after meals.
What is the single most appropriate investigation?
a. Pulmonary angiography
b. Gastroscopy
c. Cardiac catheterization
d. Oesophagoscopy
e. Indirect laryngoscopy

19. A 32 year old woman who works in a pet shop complains of fever, a dry cough and increasing
breathlessness.
What is the single most causative organism?

a. Chlamydia psittaci
b. Mycobacterium tuberculosis
c. Staphylococcus aureus
d. Haemophilus influenzae
e. Streptococcus pneumoniae

20. A 30 year old woman has just returned from a holiday in Cyprus. She complains of a dry cough,
fever, malaise. A CXR shows patchy consolidation.
What is the single most causative organism?

a. Mycoplasma pneumoniae
b. Pneumocystis carinii
c. Legionella pneumoniae
d. Klebsiella pneumoniae
e. Mycobacterium tuberculosis

21. A 34 year old man presents with history of chest pain radiating to the jaw, brought on by
exercise and relieved by rest.
What is the single most appropriate investigation?

a. CT scan
b. ECG
c. USG
d. ECHO
e. Pulmonary angiography

22. A 67 year old woman presents with a 2hr history of chest pain radiating to the left shoulder.
He is sweating and feels sick.
What is the single most appropriate investigation?

a. CXR
b. CT Scan
c. EEG
d. ECG
e. ECHO

23. A tall young man presents with history of sudden chest pain radiating to the back and
interscapular area
What is the single most appropriate investigation?

a. CT scan
b. ECG
c. USG
d. ECHO
e. Pulmonary angiography

24. A 52 year old woman on HRT presents with swollen left calf, chest pain an shortness of breath.
What is the single most likely disease process?

a. Pulmonary embolism
b. Pneumonia
c. Bronchial asthma
d. Plueral effusion
e. Cryptogenic fibrosing alveolitis

25. A 78 year old man has been short of breath for a few weeks. His chest radiography shows a
right basal shadow rising towards axilla.
What is the single most likely disease process?

a. Pulmonary embolism
b. Pneumonia
c. Bronchial asthma
d. Plueral effusion
e. Cryptogenic fibrosing alveolitis

26. A 45 year old woman presents with progressive breathlessness and cyanosis. Clinical
examination reveals clubbing and bilateral inspiratory crackles.
What is the single most likely disease process?

a. Pulmonary embolism
b. Pneumonia
c. Bronchial asthma
d. Plueral effusion
e. Cryptogenic fibrosing alveolitis
27. A 27 year old farmer presented with fever, malaise, cough and breathlessness which he has
had for a few days. His symptoms were worse in the evening. Clinical examination
demonstrated coarse end inspiratory crackles.
What is the single most likely disease process?
a. Anaemia
b. Pulmonary oedema
c. Bronchogenic carcinoma
d. Extrinsic allergic alveolitis
e. Pneumothorax

28. A 34 year old tall slim porter presents with sudden onset chest pain and breathlessness. He
has had similar episodes in the past.
What is the single most likely disease process?

a. Anaemia
b. Pulmonary oedema
c. Bronchogenic carcinoma
d. Extrinsic allergic alveolitis
e. Pneumothorax

29. A 76 year old man presents with acute breathlessness, cough, frothy blood stained sputum
and palpitations. He is apyrexial. ECG shows sinus tachycardia.
What is the single most likely disease process?

a. Anaemia
b. Pulmonary oedema
c. Bronchogenic carcinoma
d. Extrinsic allergic alveolitis
e. Pneumothorax

30. A 37 year old man presents with dyspnoea, cough, weight loss and night sweats. His CXR
shows diffuse bilateral infiltrates.
What is the single most causative organism?

a. Pneumocystis carinii
b. Chlamydia psittaci
c. E.coli
d. Pseudomonas aeruginosa
e. Mycobacterium tuberculosis

31. A 48 year old presents with fever, rigors, headache and diarrhoea. He recently had been on
holiday abroad. CXR shows consolidation.
What is the single most causative organism?

a. Staphylococcus aureus
b. Cryptococcus
c. Streptococcus pneumoniae
d. Legionella pneumophilia
e. Mycobacterium avium-intracellulare
32. A 45 year old farmer presents with flu like illness, anorexia and dry cough. His CXR shows
patchy consolidation.
What is the single most causative organism?

a. Legionella pneumophila
b. Mycobacterium avium-intracellulare
c. Mycoplasma pneumoniae
d. Pneumocystis carinii
e. Chlamydia psittaci

33. A 41 year old drug abuser presents with fever, cough and breathlessness. This was preceded
by viral influenza. Chest radiograph shows multiple abscesses.
What is the single most causative organism?

a. Pneumocystis carinii
b. Chlamydia psittaci
c. Escherichia coli
d. Pseudomonas aeruginosa
e. Mycobacterium tuberculosis

34. A 14 year old student with cystic fibrosis rapidly deteriorated developed acute respiratory
failure while in hospital.
What is the single most causative organism?

a. Cryptococcus
b. Streptococcus pneumoniae
c. Legionella pneumophila
d. Mycobacterium avium-intracellulare
e. Mycoplasma pneumoniae

35. A 35 year old smoker with a long history of recurrent chest infection presents with
haemoptysis and greenish sputum. On examination he has clubbing and coarse crepitations
over the bases of both lungs.
What is the single most likely disease process?

a. Pulmonary infarction
b. Pneumonia
c. Tuberculosis
d. Bronchiectasis
e. Mitral stenosis

36. A 69 year old woman who had a total hip replacement a week ago presents with severe chest
pain, shortness of breath and haemoptysis.
What is the single most likely disease process?

a. Pulmonary infarction
b. Pneumonia
c. Tuberculosis
d. Bronchiectasis
e. Mitral stenosis

37. A 52 year old diabetic presents with fever, pleuritic pain and rusty coloured sputum.
What is the single most likely disease process?

a. Pneumonia
b. Tuberculosis
c. Bronchiectasis
d. Mitral stenosis
e. SLE

38. A 45 year old bank manager presents with cough, pleuritic chest pain and haemoptysis. This
was preceded by rhinitis, recurrent epistaxis and haematuria, CXR shows multiple nodular
masses.
What is the single most likely disease process?

a. Foreign body inhalation


b. Wegener’s granulomatosis
c. Haemorrhagic telangiectasia
d. Polyarteritis nodosa
e. Bronchogenic carcinoma

39. A 70 year old smoker presents with cough, haemoptysis and weight loss. On examination,
there is clubbing and gynaecomastia.
What is the single most likely disease process?

a. Foreign body inhalation


b. Wegener’s granulomatosis
c. Haemorrhagic telangiectasia
d. Polyarteritis nodosa
e. Bronchogenic carcinoma

40. A 72 year old Asian immigrant presents with cough, haemoptysis, night fever and sweating.
What is the single most likely disease process?

a. Tuberculosis
b. Bronchiectasis
c. Mitral stenosis
d. SLE
e. Foreign body inhalation
41. A 50 year old man presents with progressive burning pains in the sole of his left foot, bilateral
cramps in both calves, and left foot drop. He is known asthmatic and suffers from recurrent
sinusitis. FBC shows eosinophilia.
What is the single most likely disease process?

a. Acute myocardial infarction


b. Churg-Strauss syndrome
c. Pulmonary embolism
d. Pneumothorax
e. Allergic bronchopulmonary

42. A 33 year old man presented with breathlessness at rest, cough and haemoptysis all of which
he has had for a few days. On examination he is cyanosed and there are bilateral inspiratory
and expiratory wheezes. His PEFR is normal. Blood investigations show evidence of renal
failure.
What is the single most likely disease process?

a. Extrinsic allergic alveolitis


b. Pneumocystis carinii
c. Cystic fibrosis
d. Cryptogenic fibrosing alveolitis
e. Good pastures syndrome

43. A 73 year old presents with pneumonia which he has had for two weeks and which was
resistant to antibiotics. He has had asthmatic bronchitis for more than 50 years. CXR shows
consolidation of the right upper zone and the left perhilar consolidation. Blood tests shows
neutrophilia.
What is the single most likely disease process?

a. Acute myocardial infarction


b. Churg-Strauss syndrome
c. Pulmonary embolism
d. Pneumothorax
e. Allergic bronchopulmonary

44. A 29 year old farmer presented with worsening cough breathlessness and flu like symptoms
which he had each winter for 3 years. CXR showed fine military shadows. Pulmonary function
tests showed evidence of restrictive lung disease.
What is the single most likely disease process?

a. Extrinsic allergic alveolitis


b. Pneumocystis carinii
c. Cystic fibrosis
d. Cryptogenic fibrosing alveolitis
e. Good pastures syndrome
45. A 30 year old woman presents with fever, pharyngitis and cough. The CXR shows widespread
bilateral patchy consolidation. Erythema multiform is present.
What is the single most likely disease process?

a. Mycoplasma pneumoniae
b. Bilateral bronchopneumonia
c. Adult respiratory distress syndrome
d. Sarcoidosis
e. Pulmonary contusion

46. A 40 year old alcoholic presents with repeated small haemoptysis and cough of mucoid
sputum. His CXR shows right upper lobe consolidation and a large central cavity. Heaf test is
positive.
What is the single most likely disease process?

a. Bilateral bronchopneumonia
b. Adult respiratory distress syndrome
c. Sarcoidosis
d. Pulmonary contusion
e. Tuberculosis

47. A 60 year old presents with dyspnoea and cough. The CXR shows extensive pulmonary fibrosis,
bilateral Plueral thickening and Plueral calcification.
What is the single most likely disease process?

a. Pneumoconiosis
b. Carcinoma bronchus
c. Cystic fibrosis
d. Pancoasts tumour
e. Mycoplasma pneumoniae

48. A 14 year old boy presents with repeated lower respiratory infections. On examination he is
noted to have finger clubbing and suffers weight loss and steatorrhea. The CXR shows
bronchial wall thickening, ring shadows of bronchiectasis and wide spread ill- defining
shadowing.
What is the single most likely disease process?

a. Cystic fibrosis
b. Pancoasts tumour
c. Mycoplasma pneumoniae
d. Bilateral bronchopneumonia
e. Adult respiratory distress syndrome

49. A 40 year old man presents with cough and haemoptysis. The CXR shows a right hilar mass
and a patch of consolidation in the right upper lobe laterally.
What is the single most likely process?

a. Carcinoma bronchus
b. Cystic fibrosis
c. Pancoasts tumour
d. Mycoplasma pneumoniae
e. Bilateral bronchopneumonia

50. A 20 year old woman presents with a week’s history of fever, rigors and productive rusty
cough. CXR shows right lower lobe consolidation.
What is the single most appropriate treatment?

a. Tetracycline
b. Flucloxacillin
c. Benzyl penicillin
d. Cefotaxime
e. Prednisolone

51. A 12 year old boy with cystic fibrosis presents with persisted productive cough. The CXR shows
spherical shadow containing a central lucency. An air fluid level is also seen.
What is the single most appropriate treatment?

a. Erythromycin
b. Tetracycline
c. Flucloxacillin
d. Benzyl penicillin
e. Cefotaxime

52. A 50 year old man presents with shortness of breath and dry cough. CXR shows widespread
of pulmonary shadowing. He takes azathioprine for resistant rheumatoid arthritis.
What is the single most appropriate treatment?

a. Erythromycin
b. Tetracycline
c. Flucloxacillin
d. Benzyl penicillin
e. Cefotaxime

53. A 20 year old woman presents with malaise, cough and progressive shortness of breath. The
CXR shows symmetrical lobulated bilateral hilar hand enlargement.
What is the single most appropriate treatment?

a. Tetracycline
b. Flucloxacillin
c. Benzyl penicillin
d. Cefotaxime
e. Prednisolone

54. A 50 year old diabetic presents with a productive cough and malaise. CXR shows right upper
lobe consolidation and hilar lymphadenopathy.
What is the single most appropriate treatment?
a. Rifampicin + isoniazid
b. Pyridoxine and ethambutol
c. Isoniazid alone
d. Cotrimoxazole
e. Salbutamol inhaler

55. A 35 year old pet shop owner presents with high fever, excruciating headache and a dry
hacking cough. The CXR shows patchy consolidation.
What is the single most likely causative organism?

a. Chlamydia psittaci
b. Actinomycosis
c. Pneumocystis carinii
d. Streptococcus pneumoniae
e. Mycoplasma pneumoniae

56. A 30 year old man who works in an abattoir presents with sudden onset of fever, myalgia,
headache, dry cough and chest pain. CXR shows patchy consolidation of the right lower lobe
giving a ground glass appearance.
What is the single most likely causative organism?

a. Tuberculosis
b. Pseudomonas aeruginosa
c. Coxiella burnetti
d. Legionella pneumophilia
e. Aspergillosis

57. A 44 year old travelling insurance salesman presents with high fever, myalgia, abdominal pain,
and haemoptysis. CXR shows diffuse patchy lobar shadows. The cough progresses from a
modest non productive cough to producing mucopurulent sputum. The fever persists for 2
weeks.
What is the single most likely causative organism?

a. Tuberculosis
b. Pseudomonas aeruginosa
c. Coxiella burnetti
d. Legionella pneumophilia
e. Aspergillosis

58. A 10 year old man with HIV presents with a productive cough and haemoptysis. CXR shows a
round ball in the right upper lobe super mounted by a dome of air.
What is the single most likely causative organism?
a. Tuberculosis
b. Pseudomonas aeruginosa
c. Coxiella burnetti
d. Legionella pneumophilia
e. Aspergillosis
59. A 20 year old male IV drug abuser presents with breathlessness and cough. CXR reveals patchy
areas of consolidation with abscess formation.
What is the single most likely causative organism?

a. Streptococcus pneumoniae
b. Mycoplasma pneumoniae
c. Haemophilus influenzae
d. Staphylococcus aureus
e. Legionella pneumophila

60. A 12 year old boy with cystic fibrosis presents with a chest infection. The boy also suffers from
mild renal failure.
What is the single most appropriate treatment?

a. Tobramycin and carbenicillin


b. Plasmapheresis
c. Ciprofloxacin
d. Tetracycline
e. Cotrimoxazole

61. A 30 year old presents with haemoptysis, dyspnoea and haematuria. CXR reveals bilateral
alveolar infiltrates. Urine analysis reveals presence of protein and red cell casts.
What is the single most appropriate next investigation?

a. 12 lead ECG
b. Tissue biopsy
c. ANA
d. Sputum cytology
e. Antiglomerular basement antibodies

62. A 60 year old man presents with a chronic cough and mild haemoptysis. On examination he
has digital clubbing with pain and swelling around his wrists. CXR reveals a single nodule.
What is the single most appropriate next investigation?

a. Pulmonary angiogram
b. 12 lead ECG
c. Tissue biopsy
d. ANA
e. Sputum cytology

63. A 30 year old IV drug abuser presents with dyspnoea and haemoptysis. His CXR is
unremarkable and ECG shows sinus tachycardia with a mean paxis shift to the right. ABG
shows a low PaCO2 and an elevated pH.
What is the single most appropriate next investigation?

a. Heaf test
b. Bronchography
c. Urinalysis
d. Chest x-ray
e. Pulmonary angiogram

64. A 40 year old mental patient with dry cough and confusion. Blood tests reveal lymphopenia
and hyponatremia. CXR shows right lobar shadowing.
What is the single most appropriate treatment?

a. Erythromycin
b. Cyclophosphamide
c. Tobramycin and carbenicillin
d. Plasmapheresis
e. Ciprofloxacin

65. A 10 year old boy presents with wheezing attacks and episodic shortness of breath. His PEFR
is 400 L/min.
What is the single most appropriate treatment?

a. Beta 2 agonist
b. Rifampicin + isoniazid
c. IV aminophylline
d. Prednisolone
e. Erythromycin

66. A 40 year old male presents with rhinorrhoea, cough, haemoptysis and pleuritic chest pain.
CXR shows multiple modules.
What is the single most appropriate treatment?

a. Erythromycin
b. Cyclophosphamide
c. Tobramycin and carbenicillin
d. Plasmapheresis
e. Ciprofloxacin

67. A 60 year old farmer presents with fever, cough and shortness of breath. He had been forking
hay that morning. CXR shows fatty n=nodular shadows in the upper zone.
What is the single most appropriate treatment?
a. Prednisolone
b. Erythromycin
c. Cyclophosphamide
d. Tobramycin and carbenicillin
e. Plasmapheresis

68. A 440 year old man presents with recurrent epitaxial, haemoptysis and haematuria. On
examination he has nasal septai perforation and nodules on CXR.
What is the single most appropriate next investigation?

a. Pulmonary angiogram
b. 12 lead ECG
c. Tissue biopsy
d. ANA
e. Sputum cytology

69. A 50 year old man presents with occasional haemoptysis and chronic productive cough. He
has a history of recurrent pneumonia. CXR reveals peri bronchial fibrosis.
What is the single most appropriate next investigation?

a. FBC
b. CT chest
c. Clotting studies
d. Heaf test
e. Bronchography

70. 30 year old woman: ABG- pH 7.51, pCO2 3.7 KPa, pO2 14.3 KPa, HCO3- 28mmol/l.
What is the single most likely diagnosis?

a. Pulmonary embolism
b. Panic attack
c. Sarcoidosis
d. Cystic fibrosis
e. Extrinsic tracheal compression

71. A 40 year old afro Caribbean woman: PFT-FEV12.51,FVC2.91,FEV1/FVC86%,serumACElevels-


high
What is the single most likely diagnosis?

a. Pneumonectomy
b. Pulmonary fibrosis
c. Pulmonary embolism
d. Panic attack
e. Sarcoidosis

72. A 60 year old male non-smoker: PFTs- FEV 1.14L, FVC 3.51, FEV1/FVC = 40%. After
bronchodilator trial- FEV1/FVC = 50%. After 2 weeks of prednisolone 30 mg daily REV1/FVC =
65%.
What is the single most likely diagnosis?

a. Asthma
b. Pulmonary haemorrhage
c. Emphysema
d. Extrinsic allergic alveolitis
e. Chronic bronchitis

73. A 29 year old teacher is 7 months pregnant. She presents with sudden collapse and
breathlessness. On examination, she is afebrile, severely cyanosed and hyperventilating. Her
pulse is 140/min and BP 70/30 mmHg.
What is the single most appropriate next investigation?

a. CT brain
b. Blood count and film
c. Spiral CT with contrast
d. ECHO
e. Arterial blood gases

74. A 74 year old woman presents to causality with acute breathlessness. She has had 3 similar
admissions in last few months. She has a past history of rheumatic fever. On examination, she
has an irregular pulse of 110/min. she has a non-displaced tapping apex beat with no evidence
of left ventricular dysfunction. No murmurs are audible. There are crepitation’s heard in both
lower lung fields.
What is the single most likely cause of pulmonary oedema?

a. Adult respiratory distress syndrome


b. Mitral stenosis
c. Anaemia
d. Aortic stenosis
e. Acute renal failure

75. A 67 year old has had an elective total knee replacement. He has no significant medical history.
36 hrs after operation he complains of increasing breathlessness. He is cyanosed and has
crackles up to mid zones of both lung fields. His ECG is normal cardiac dimensions. His urine
output since the operation has been poor and e has been persistently hypotensive with a BP
of around 90/60 mmHg.
What is the single most likely cause of pulmonary oedema?

a. Myocardial infarction
b. Constrictive pericarditis
c. Myocarditis
d. Iatrogenic
e. Cardiomyopathy

76. A 50 year old farmer PFTs: FEV1 2.81, FVC 3.51, FEV1/FVC 80%. Reduced TLCO and KC Broncho
alveolar lavage – lymphocytes and mast cells. Precipitins to microspore faeni – positive.
What is the single most likely diagnosis?

a. Emphysema
b. Extrinsic allergic alveolitis
c. Chronic bronchitis
d. Pneumonectomy
e. Pulmonary fibrosis

77. A 35 year old woman ABG: pH 7.4, pCO2 4.4, pO2 9.4, TLCO 90% of predicted, KCO160% of
predicted. All results were normal 24 hours ago.
What is the single most likely diagnosis?
a. Pulmonary haemorrhage
b. Emphysema
c. Extrinsic allergic alveolitis
d. Chronic bronchitis
e. Pneumonectomy

78. A 26 year old nurse is brought to casualty as an emergency. She is hyperventilating but drowsy,
she has been complaining of nausea and tinnitus and had an episode of hematemesis.
What is the single most appropriate next investigation?

a. Salicylate levels
b. Serum lactose
c. CT brain
d. Blood count and film
e. Spiral CT with contrast

79. A 21 year old motorcycle courier was involved in a head on collision with a van. He had an
obvious deformity of his left thigh and knee. X- Ray confirmed communited fracture of the
femur that was reduced and placed in traction. Five days later he suddenly deteriorates
becomes drowsy, confused and febrile. He is hyperventilating and cyanosed. There are
crackles at both lung bases and petechiae over his chest and neck.
What is the single most appropriate next investigation?

a. Blood count and film


b. Spiral CT with contrast
c. ECHO
d. Arterial blood gases
e. V/Q scan

80. A 15 year old school girl was out with her friends at a party last night. When she returned
home this morning she was drowsy unwell and vomiting. She has lost 5kg over last 2 months.
On arrival at hospital, she is drowsy, confused and hyperventilating but not cyanosed. Her
breath smells of pear drops.
What is the single most appropriate next investigation?

a. Blood glucose
b. Salicylate levels
c. Serum lactose
d. CT brain
e. Blood count and film

81. A 24 year old car mechanic is brought to casualty by his girlfriend. She describes a 2 day history
of rigors, sweats and intermittent confusion. On examination he is agitated, sweaty and
pyrexial with 38.6°C temp. He is hyperventilating and cyanosed despite receiving O2 by face
mask. There is dullness to percussion and bronchial breathing at the left lung base.
What is the single most appropriate next investigation?
a. None required
b. Urea and electrolytes
c. Chest X-ray
d. Blood glucose
e. Salicylate levels

82. A 14 year old boy presents to casualty complaining of severe chest pain and difficulty in
breathing. He is hyperventilating and pale but not cyanosed. He has had 4 similar admissions
in last year and his older brother also attends the hospital frequently.
What is the single most appropriate next investigation?

a. Blood glucose
b. Salicylate levels
c. Serum lactose
d. CT brain
e. Blood count and film

83. A 21 year old man has become increasingly breathlessness over a period of 4 days. He has also
had severe central chest pain and a fever. He was previously fit and well. On examination, he
looks unwell, cyanosed and dyspnoeic. Pulse is 120/min, regular BP 90/40 mmHg. JVP elevated
and he has a gallop rhythm with no murmurs. These are crackles in both long bases. CXR shows
pulmonary oedema and a normal heart rate. ECG shows extensive ST elevation in the anterior
and inferior leads without Q waves.
What is the single most likely cause of pulmonary oedema?

a. Myocarditis
b. Iatrogenic
c. Cardiomyopathy
d. Hypoalbuminemia
e. Mitral regurgitation

84. A 38 year old woman was admitted to hospital two days ago with abdominal pain and
vomiting. She was tender in the epigastrium and was found to have a very serum amylase
level. She has been treated with large volumes of IV fluids and has maintained a good urine
output. Nonetheless she has been persistently hypoxic and is deteriorating rapidly. She now
has crackles throughout both lung fields and a pO2 of 5.1kPa despite receiving 60% O2 by
mask. CXR shows massive bilateral pulmonary oedema. Her serum albumin is 30 mmol/l.
What is the single most likely cause of pulmonary oedema?

a. Adult respiratory distress syndrome


b. Mitral stenosis
c. Anaemia
d. Aortic stenosis
e. Acute renal failure

85. A 30 year old lady complains increasing breathlessness, tiredness, malaise, poor appetite and
weight loss.
What is the single most likely cause of breathlessness?
a. Asthma
b. Tuberculosis
c. Angioneurotic oedema
d. Thyroiditis
e. Branchial cyst

86. A 56 year old man wheezes and coughs. He has tried to give up smoking, but he finds it very
difficult. He is thin and healthy looking with a rounded chest. His breathing is noisy. His cough
is unproductive.
What is the single most appropriate next step in management?

a. Ipratropium
b. Amoxicillin
c. Aminophylline
d. Prednisolone
e. Salbutamol

87. A 37 year old man who has had recurrent chest infections since a serious bout of influenza 3
years ago presents with chronic productive cough. His sputum is tenacious and blood stained,
on auscultation you find crackling.
What is the single most appropriate next step in management?

a. Postural drainage
b. Aspiration
c. Chest wall drain
d. Bronchoscopy
e. Pluerectomy

88. A woman has been treated for hypothyroidism cannot breathe. At night she must sit propped
up and she has a constant dull chest pain. On examination of her chest you find stony dullness
and bronchial breathing.
What is the single most appropriate next step in management?

a. Postural drainage
b. Aspiration
c. Chest wall drain
d. Bronchoscopy
e. Pluerectomy

89. A healthy young woman comes to you complaining of a swelling in front of the next and
diarrhoea. She also has palpitations and is short of breath.
What is the single most likely cause of breathlessness?

a. Asbestosis
b. CML
c. Erysipelas
d. Primary thyrotoxicosis
e. Hay fever

90. A 50 year old train conductor complains of chronic productive cough. He is also a heavy
smoker.
What is the single most likely diagnosis?

a. Erysipelas
b. Primary thyrotoxicosis
c. Hay fever
d. Depression
e. Bronchogenic carcinoma

91. A 14 year old girl is brought into A and E with very difficult and noisy breathing.
What is the single most likely cause of breathlessness?

a. Bronchogenic carcinoma
b. Sinusitis
c. Asthma
d. Tuberculosis
e. Angioneurotic oedema

92. A young woman complains of sore eyes, a dull chest pain, malaise and a low grade fever. She
thinks she has had a recurrent low grade fever for 4 months. A CXR shows bilateral hilar
enlargement.
What is the single most likely diagnosis?

a. Sarcoidosis
b. Asthma
c. Histiocytosis
d. Emphysema
e. Aspergillus funigatus

93. A middle aged pharmaceutical engineer presents with increasing breathlessness and cough.
He doesn’t smoke. On CXR you find diffuse bilateral mottling and multiple small cystic lesions.
Six months ago he had a pneumothorax.
What is the single most likely diagnosis?

a. Sarcoidosis
b. Asthma
c. Histiocytosis
d. Emphysema
e. Aspergillus funigatus
94. For the last three years a young man has complained of recurrent bouts of pneumonia with
wheeziness, cough, fever and malaise. His sputum is tenacious. Peripheral blood shows a very
high ESR and Ige.
What is the single most likely diagnosis?

a. Aspergillus fumigatus
b. Lung fibrosis
c. Idiopathic thrombocytopenia
d. Pleurisy
e. Polyarteritis nodsa

95. A 30 year old computer engineer with a long history of asthma and rhinitis presents with
wheezing, cough and fever. A CXR shows patchy consolidation. Physical examination shows
multiple tender subcutaneous nodules and purpura.
What is the single most likely diagnosis?

a. Pleurisy
b. Polyarteritis nodosa
c. Lung collapse
d. Hyperparythyroidism
e. Tuberculosis

96. A 43 year old man presents with chest pain and cough. He complains of fullness pressure in
the chest and a sharp pain affecting 2 or 3 ribs on the right side. A CXR shows a right sided
hilar enlargement.
What is the single most likely diagnosis?

a. Lung collapse
b. Hyperparathyroidism
c. Tuberculosis
d. Silent MI
e. Bronchogenic carcinoma

97. A middle aged woman brings her husband into your surgery. She complains of his heavy
snoring. Also in the middle of the night he stops breathing, gags and wakes up. Also his
performance at work has been declining and he is being threatened of retirement.
What is the single most appropriate next step in management?

a. Surgical excision
b. CPAP
c. Ipratropium
d. Amoxicillin
e. Aminophylline
98. A young man with chronic hepatitis develops chest complaints. He has become breathless and
coughs. He has lost weight and his joints bother him. On physical examination you note mild
cyanosis.
What is the single most appropriate step to take in management?

a. Ipratropium
b. Amoxicillin
c. Aminophylline
d. Prednisolone
e. Salbutamol

99. A middle aged woman complains of dyspnoea. Over the past year she had to give up her
gardening. An ECG shows right ventricular hypertrophy and strain.
What is the single next appropriate step in management?

a. Lobectomy
b. Antidepressants
c. A healthy diet
d. Anticoagulants
e. Contraception

100. An elderly woman cannot carry on her household chores but has to sit down every few minutes.
A CXR shows diffuse reticular nodular shadows especially in her lower lobes.

What is the single next most appropriate step in management?

a. Oxygen therapy
b. Thymectomy
c. Chemotherapy
d. Change occupation
e. Radiotherapy

101. A middle aged housewife presents with exertional dyspnoea. She also has a dry cough, general
malaise and is mildly cyanosed. On auscultation of chest you find crepitations. She has a history of
arthritis.

What is the single most likely diagnosis?

a. Respiratory failure
b. Bronchitis
c. Bronchial asthma
d. Emphysema
e. Rheumatoid lung

102. A 35 year old accountant presents with a chronic cough, dysponea and wheezing. He produces
copious sputum. His arterial carbon dioxide is low and his arterial oxygen is normal.

What is the single most likely diagnosis?

a. Rheumatic heart disease


b. Respiratory failure
c. Bronchitis
d. Bronchial asthma
e. Emphysema

103. A middle aged smoker presents with chronic cough and phlegm. His sputum is tenacious but
not yellow or blood stained. His chest is hyper inflated. His arteria carbon dioxide is high and his
arterial oxygen is low.

What is the single most likely diagnosis?

a. Bronchitis
b. Bronchial asthma
c. Emphysema
d. Rheumatoid lung
e. Fibrosing alveolitis

104. A 53 year old smoker with chronic cough and copious yellow sputum presents in a state of
agitation. He is confused. His pulse is bounding. He has a terrible headache and you find papilledema
on fundoscopy.

What is the single most likely diagnosis?

a. Tuberculosis
b. Pulmonary embolism
c. Rheumatic heart disease
d. Respiratory failure
e. Bronchitis

105. A young woman complains of wheeze, dyspnoea and cough. She cannot sleep at night because
of a chronic cough. She and her mother love animals and together they have 14 cats. Her PEFR is
normal but her CXR suggests hyperinflation.

What is the single most likely diagnosis?

A. Respiratory failure
B. Bronchitis
C. Bronchial asthma
D. Emphysema
E. Rheumatoid lung

106. A 36 year old woman presents “with increasing breathlessness over a period of 10 months. She
now finds herself unable to climb stairs. Also she has just had her 3rd child in 3 years and complains
of feeling depressed.

What is the single next most appropriate step in management?

A. Blood transfusion
B. Lobectomy
C. Antidepressants
D. A healthy diet
E. Anticoagulants

107. A 43 year old shipyard worker complains of gasping for breath. On physical examination you
discover mild cyanosis and restricted chest movements.
What is the single next most appropriate step in management?

A. Oxygen therapy
B. Thymectomy
C. Chemotherapy
D. Change occupation
E. Radiotherapy

108. A 62 year old man says he can’t get enough air. Over the past few months he has become
increasingly breathless. Also, he has noticed he has had difficulty in finishing his dinner. His jaw tires.

What is the single next most appropriate step in management?

A. Edrophonium chloride
B. Iron supplements
C. Oxygen therapy
D. Thymectomy
E. Chemotherapy

109. A 65 year old warehouse man is admitted with severe wheeze and breathlessness. He has had
to sleep propped up. And during the past few days his legs have become swollen.

What is the single next most appropriate step in management?

A. Furosemide
B. Oxygen therapy
C. GTN
D. Morphine
E. Nasal IPPV

110. A 50 year old taxi driver and heavy smoker complains that his feet swell up at the end of the
day. He has also noticed increasing shortness of breath.

What is the single next most appropriate step in management?

A. sputum culture
B. Pericardial aspiration
C. Streptokinase
D. Furosemide
E. Oxygen therapy

111. A middle ages woman complains of a chronic cough and purulent sputum. Recently she has
become very wheezy and dyspnoeic. An ECG gives a low voltage trace and a CXR shows a large heart
shadow.

What is the single next most appropriate step in management?

A. Antibiotics
B. Sputum culture
C. Pericardial aspiration
D. Streptokinase
E. Furosemide
112. An elderly woman complains of acute chest pain, breathlessness and haemoptysis. A month ago
she had pain and swelling in the legs but did not go to the doctor with these complaints.

What is the single next most appropriate step in management?

A. Antibiotics
B. Sputum culture
C. Pericardial aspiration
D. Streptokinase
E. Furosemide

113. His friends bring in a 48 year old businessman. He is very distressed, breathless, cold and
clammy. He has severe chest pain and he has vomited 3 times.

What is the single next most appropriate step in management?

A. Morphine
B. Nasal IPPV
C. Venesection
D. Advise stop smoking
E. Beta agonists

114. A 4 year old child has severe cough and fever. She appears acutely ill and cyanosed.

What is the single most likely diagnosis?

A. Bronchogenic carcinoma
B. Parainfluenzae
C. Respiratory syncytial virus
D. Rhinovirus
E. Pleurisy

115. A 6 year old child complains of a very sore throat. He has a mild fever and red fauces.

What is the single most likely diagnosis?

A. Bronchogenic carcinoma
B. Parainfluenzae
C. Respiratory syncytial virus
D. Rhinovirus
E. Pleurisy

116. A 32 year old bus driver complains of a persistent cough and blood stained sputum. He does not
smoke. He has a fever and supraclavicular glands. A CXR shows a dense irregular hilar expansion.

What is the single most likely diagnosis?

A. Tuberculosis
B. MI
C. Asthma
D. Meningitis
E. Bronchiectasis
117. A 32 year old lecturer has had a fever for a week with no cough. Her fever came on suddenly
and she says half her department is similarly ill. She has fine rales and a positive chest X-ray.

What is the single most likely diagnosis?

A. Parainfluenzae
B. Respiratory syncytial virus
C. Rhinovirus
D. Pleurisy
E. Pneumococci

118. A 40 year man complains of swollen legs and breathlessness. Examination reveals
hepatosplenomegaly. And a chest X-ray shows a normal but dense heart shadow and clear lung
fields.

What is the single next most appropriate step in management?

A. Pericardial resection
B. Radiotherapy
C. Furosemide
D. Anti-emetics
E. Aspirin

119. A middle ages decorator complains of severe chest pain radiating to the neck. The pain came on
after dinner as he was retiring for the night.

What is the single next most appropriate step in management?

A. Diamorphine
B. Diazepam
C. Combination antibiotics
D. Anxiolytics
E. Lobectomy

120. An elderly housewife was operated on for gallstones and at operation a gangrenous gallbladder
was removed. A couple of days later she complained of nausea and feeling unwell.

What is the single next most appropriate step in management?

A. Diamorphine
B. Diazepam
C. Combination antibiotics
D. Anxiolytics
E. Lobectomy

121. A 62 year old lifelong smoker has a dry cough, weight loss and haemoptysis. CXR and CT show
right middle lobe collapse. He is sent to chest clinic for investigation.

What is the single next most appropriate investigation?

A. Bronchoalveolar lavage
B. ECHO
C. Serum precipitins
D. Fibreoptic bronchoscopy
E. Flow volume loop

122. A 54 year old farmer with intermittent dry cough, fever and dyspnoea, progressive weight loss
and fine crepitations on auscultation.

What is the single next most appropriate investigation?

A. Serum precipitins
B. Fibreoptic bronchoscopy
C. High resolution CT
D. Mantoux test
E. Seek immediate expertise help

123. A 24 year old male develops sudden onset dyspnoea, chest pain on inspiration and haemoptysis
whilst straining at stool. He had a cholecystectomy 10 days previously. He rapidly becomes cyanosed
and loosed consciousness. On examination his blood pressure is 75/50, heart rate 156 and thread
with a loud pulmonary heart sound. His jugular venous pressure is raised by 6cm.

What is the single next most appropriate investigation?

A. Seek immediate expertise help


B. Arterial blood gases
C. EGG, blood gases and V / Q scan
D. Serum calcium, ACE levels
E. Static lung volumes

124. A 22 year old barman presents with a dry cough of sudden onset. He complains of a chest pain
and rusty sputum. He also has a very high fever, rapid breathing, cyanosis and crepitations.

What is the single most likely diagnosis?

A. Pleurisy
B. Pneumococci
C. Pleural effusion
D. Q fever
E. Chronic bronchitis

125. A young officer manager complains of palpitations for 2 months. He says that when he mounts
the stairs he can feel his heart beating. His health history is unremarkable.

What is the single next most appropriate step in management?

A. Combination antibiotics
B. Anxiolytics
C. Lobectomy
D. Pericardial tap
E. Incision and drainage

126. A 37 year old Nigerian woman develops clubbing, erythema nodosum, iritis, dyspnoea and
lymphadenopathy on her CXR.

What is the single next most appropriate investigation?

A. Seek immediate expertise help


B. Arterial blood gases
C. EGG, blood gases and V / Q scan
D. Serum calcium, ACE levels
E. Static lung volumes

127. A 47 year old Caucasian childminder in Glasgow presents with dry cough, night sweats and
haemoptysis.

What is the single next most appropriate investigation?

A. Flow volume loop


B. Mantoux test
C. Seek immediate expertise help
D. Arterial blood gases
E. EGG, blood gases and V / Q scan

128. What is the single most characteristic lung function test picture in emphysema?

A. FEV1/FVC 62% no reversibility to bronchodilators, gas trapping, hyperinflation and reduced


transfer factor
B. Normal PFTs
C. FEV1/FVC 89%
D. FEV1/FVC 85% globally reduced lung vols. Reduced inspiratory and expiratory mouth
pressures
E. Reduced transfer factor alone

129. What is the single most characteristic lung function test picture in pneumonectomy?

A. FEV1?FVC 85% globally reduced lung vols. Reduced inspiratory and expiratory mouth
pressures
B. Reduced transfer factor alone
C. Inspiratory and expiratory fixed obstruction on volume loop
D. Spirometry, lung volumes and incorrected transfer factor 50% predicted. Corrected factor
normal
E. Increased transfer factor FEV1?FVC 58%, 25% reversibility to bronchodilators, normal
transfer factor

130. What is the single most characteristic lung function test picture in neuromuscular disease?

A. FEV1/FVC 85% globally reduced lung vols. Reduced inspiratory and expiratory mouth
pressures
B. Reduced transfer factor alone
C. Inspiratory and expiratory fixed obstruction on volume loop
D. Spirometry, lung volumes and incorrected transfer factor 50% predicted. Corrected factor
normal
E. Increased transfer factor FEV1?FVC 58%, 25% reversibility to bronchodilators, normal
transfer factor

131. What is the single most characteristic lung function test picture in interstitial lung disease?

A. FEV1/FVC ratio 90%, TLC reduced by 30%, considerably reduced transfer factor
B. FEV1/FVC 62%, no reversibility to bronchodilators, gas tapping, hyperinflation and reduced
transfer factor
C. Normal PFTs
D. FEV1/FVC 89%
E. FEV1/FVC 85%, globally reduced lung vols. Reduced inspiratory and expiratory mouth
pressures

132. A 30 year old woman presents with a 3 day history of pyrexia, rigors and sweats. This was
preceded by a 1 week history of cough and increasing dysponea. She had a swinging fever and
dullness to percussion at the left lung base.

What is the single most likely cause of pleural effusion in this case?

a. Emphysema
b. Rheumatoid arthritis
c. Tuberculosis
d. SLE
e. Bronchial carcinoma

133. A 12 year old female presents with increasing breathlessness and abdominal swelling. She has a
right sided Plueral effusion. On examination she has generalised lymphadenopathy.

What is the single most likely cause of pleural effusion in this case?

a. Rheumatoid arthritis
b. Tuberculosis
c. SLE
d. Bronchial carcinoma
e. Yellow nail syndrome

134. A 31 year old drug abuser presents with a 3 week history of cough, fever and general malaise
with occasional haemoptysis. He has a right sided pleural effusion.

What is the single most likely cause of pleural effusion in this case?

a. Pneumonia
b. Pleural metastases
c. Emphysema
d. Rheumatoid arthritis
e. Tuberculosis

135. A 55 year old retired builder has a 3 month history of cough and left sided chest pain. He has a
pleural effusion on the left. On CXR there is evidence of pleural calcification.

What is the single most likely cause of pleural effusion in this case?

a. Cardiac failure
b. Mesothelioma
c. Hypoalbuminemia
d. Meig’s syndrome e
e. Pneumonia

136. A 25 year old woman has just returned from a holiday in Kenya. She suddenly became
breathless and is cyanosed. Her pulse is 120/min, BP 110/70 mmHg, PEFR 400 L/min. CXR is normal.
What is the single most next appropriate step in management?

a. IV insulin
b. IV adrenaline
c. Rebreathing into paper bag
d. Heparin
e. Methylene blue

137. A 50 year old with pulmonary fibrosis develops sudden left sided pleuritic pain and dysponea.
He has reduced air entry in the left side of the chest and percussion is hyper resonant. O2 saturation
is 80%.

What is the single most next appropriate step in management?

a. Chest drain
b. IV furosemide
c. Rapid infusion of saline
d. Forced alkaline diuresis
e. Intramuscular adrenaline

138. A 58 year old woman presents with intermittent breathlessness and couihg. On examination,
she has a right sided pleural effusion and discoloured nails.

What is the single most likely cause of pleural effusion in this case?

a. Rheumatoid arthritis
b. Tuberculosis
c. SLE
d. Bronchial carcinoma
e. Yellow nail syndrome

139. A 72 year old woman presents with a 2 month history if increasing breathlessness and swollen
legs. On examination, she has a raised JVP and a left sided pleural effusion.

What is the single most likely cause of pleural effusion in this case?

a. Cardiac failure
b. Mesothelioma
c. Hypoalbuminemia
d. Meig’s syndrome e
e. Pneumonia

140. A pet shop owner presents with high swinging fever, cough, and malaise. He has scanty rose
spots over his abdomen. CXR reveals diffuse pneumonia.

What is the single most likely causative organism?

a. Chlamydia psittaci
b. Streptococcus pneumoniae
c. Mycoplasma pneumoniae
d. Haemophilus influenzae
e. Staphylococcus aureus
141. A 70 year old woman presents with confusion and productive cough. CXR shows right lower
lobe consolidation.

What is the single most likely causative organism?

a. Actinomycosis
b. Pneumocystis carinii
c. Streptococcus pneumoniae
d. Mycoplasma pneumoniae
e. Staphylococcus aureus

142. A 70 year old alcoholic man presents with sudden onset of purulent productive cough. CXR
shows consolidation left upper lobe.

What is the single most likely causative organism?

a. Pneumocystis carinii
b. Aspergillus fumigatus
c. CMV
d. Actinomyces israelii
e. Klebsiella pneumoniae

143. A 10 year old boy with cystic fibrosis presents with pneumonia.

What is the single most likely causative organism?

a. Pseudomonas aeruginosa
b. Pneumocystis carinii
c. Aspergillus fumigatus
d. CMV
e. Actinomyces israelii

144. A 30 year old man with AIDS presents with fever, dry cough and dysponea, CXR shows diffuse
bilateral alveolar and interstitial shadowing beginning in the perihilar regions and spreading
outward.

What is the single most likely causative organism?

a. Pseudomonas aeruginosa
b. Pneumocystis carinii
c. Aspergillus fumigatus
d. CMV
e. Actinoyces israelii

145. What is the best criterion for the diagnosis of asthma?

a. 10% reversibility
b. 5% reversibility
c. 15% reversibility
d. FVC
e. Positive pollen skin test

146. A 9 year old boy has a mild cough and wheeze after playing football in the cold weather.
What is the single most appropriate treatment option?

a. Oral theophylline
b. Inhaled steroids
c. Regular inhaled bronchodilation
d. Intermittent inhaled bronchodilation
e. Regular oral bronchodilation

147. A 6 year old girl with asthma uses her bronchodilator twice a day to relieve her mild wheeze.
Her parents refuse to give her any treatment containing steroids.

What is the single most appropriate treatment option?

a. Desensitization
b. Oral steroids
c. Inhaled long acting bronchodilator
d. Nebulized bronchodilator
e. Inhaled sodium cromoglycate

148. A 9 year old with chronic asthma presents to A and E with rapidly worsening wheeze not
relieved with inhaled Bronchodilator steroids have been given orally.

What is the single most appropriate treatment option?

a. Nebulized bronchodilator
b. Inhaled sodium cromoglycate
c. Oral theophylline
d. Inhaled steroids
e. Regular inhaled bronchodilation

149. A 4 year old boy with eczema and recurrent wheeze whenever he gets a viral infection has now
developed night cough. There has been no improvement in spite of using inhaled bronchodilator
twice each night.

What is the single most appropriate treatment option?

a. Inhaled sodium cromoglycate


b. Oral theophylline
c. Inhaled steroids
d. Regular inhaled bronchodilation
e. Intermittent inhaled bronchodilation

150. A 14 year old boy with well controlled asthma, using inhaled steroids and a bronchodilator
comes to A and E with breathlessness and swollen lips after eating peanut butter sandwich.

What is the single most appropriate treatment option?

a. As required bronchodilator
b. IV aminophylline
c. Adrenaline intramuscularly
d. Milk free diet
e. Desensitization
151. A 10 year old boy is getting very poor grades at school and according to the headmaster seems
to think like a 2 year old. The mother also says that he has a very short temper. On examination
unusually large testes are found. The boys elder brother and uncle had similar complaints.

What is the single most likely diagnosis?

a. Fetal alcohol syndrome


b. Normal finding
c. Familial predisposition
d. Fragile X syndrome
e. Tay-sachs’ disease

152. A 5 year old girl was born after a normal delivery has been developing normally. After an acute
illness a regression of milestones has been noticed.

What is the single most likely diagnosis?

a. Birth asphyxia
b. Meningoencephalitis
c. Duchene’s muscular dystrophy
d. Klimefelter’s syndrome
e. Celiac disease

153. A 2 year old girl was born weighing 4kg after a labour that lasted 15hrs in a mother of 2
children. She is able to stand but is yet to walk. The mother says her other child had a similar history.

What is the single most likely diagnosis?

a. Birth asphyxia
b. Meningoencephalitis
c. Duchene’s muscular dystrophy
d. Klimefelter’s syndrome
e. Celiac disease

154. A 25 year old bartender gives birth to a 2.9kg baby boy. The baby is found to have a head
circumference of 35cm. she has had her job for last 6 years.

What is the single most likely diagnosis?

a. Celiac disease
b. Phenylketonuria
c. Fetal alcohol syndrome
d. Normal finding
e. Familial predisposition

155. A 5 year old boy presents with a painful knee joint. His mother notices that he has started to
limp. On examination all the movements of the hip joint are limited. X-ray of the femoral head
shows patchy density.

What is the single most likely diagnosis?

a. Transient synovitis (irritable hip)


b. Ankylosing spondylitis
c. Slipped upper femoral epiphyses
d. Perthes’ disease
e. Congenital hip dislocation

156. A 3 year old girl is brought to hospital by an anxious mother, who says the child is taking too
long to walk normally. On examination her perinem appears wide and the lumbar lordosis appears
to be increased. She complains of occasional hip pain and is seen to have a waddling gait.

What is the single most likely diagnosis?

a. Slipped upper femoral epiphyses


b. Perthes’ disease
c. Congenital hip dislocation
d. Septic arthritis
e. Juvenile rheumatoid arthritis

157. A 5 year old presents with painful right hip and is seen to be limping. Ther is no history of
trauma. The SHO in-charge admits the boy and 24hrs later, the boy has no complains and is
discharged. X-ray of the right hip appears normal and no other joints are involved.

What is the single most likely diagnosis?

a. Transient synovitis (irritable hip)


b. Ankylosing spondylitis
c. Slipped upper femoral epiphyses
d. Perthes’ disease
e. Congenital hip dislocation

158. A 3 year old girl with a running nose, fever and loss of appetite, presents to hospital. This was
followed by stridor and cough described by mother as ‘barking’

What is the single most likely diagnosis?

a. Laryngotracheobronchitis
b. Mediastinal tumour
c. Foreign body
d. Recurrent laryngeal nerve paralysis
e. Largynomalacia

159. A 4 year old boy presents to the A and E with sever difficulty in breathing and stridor. He was
febrile and had a pulse of 200/min. throat cultures done upon arrival revealed a growth of influenza
type b.

What is the single most likely diagnosis?

a. Foreign body
b. Recurrent laryngeal nerve paralysis
c. Laryngomalacia
d. Acute epiglottitis
e. Laryngeal stenosis

160. A 2 day old breastfed male infant is vomiting after each feed. Abdominal X- ray demonstrated a
“double bubble”.

What is the single most likely diagnosis?


a. Duodenal atresia
b. Pancreatitis
c. Gastro-esphageal reflux
d. Psychogenic vomiting
e. Gastroenteritis

161. A 6 week old breast fed boy has had projectile vomiting after every feed for the pat 2 weeks. He
is now lethargic, dehydrated and tachypneic.

What is the single most likely diagnosis?

a. Pyloric stenosis
b. Meconium ileus
c. UTI
d. Meningitis
e. Whooping cough

162. A 4 month old boy who is thriving has persistent vomiting, which is occasionally blood stained
and is associated with crying.

What is the single most likely diagnosis?

a. Duodenal atresia
b. Pancreatitis
c. Gastro-esphageal reflux
d. Psychogenic vomiting
e. Gastroenteritis

163. An 8 year old girl shows signs of moderated dehydration. She has vomited all fluids for 24 hrs
and the vomit is not bile stained. Her abdomen is now soft and non-tender. She has had 2 similar
episodes in the past year.

What is the single most likely diagnosis?

a. Mesenteric adentitis
b. Cyclical vomiting
c. Overfeeding
d. Duodenal atresia
e. Pancreatitis

164. A 12 week old thriving baby is vomiting after every feed. He is developmentally normal and is
fed by the bottle at 260 ml/hg day.

What is the single most likely diagnosis?

a. Mesenteric adentitis
b. Cyclical vomiting
c. Overfeeding
d. Duodenal atresia
e. Pancreatitis

165. A 15 year old boy has very poor grades at school despite being very attentive and hard working.
His mother reckons its becausehe is teased at school because his breasts look like a girls. On further
examination he is found to have a small testes. He is mildly asthmatic.
What is the single most likely diagnosis?

a. Klinefelters syndrome
b. Sickle cell tarit
c. Patau syndrome
d. Beta thalassemia
e. Downs syndrome

166. A 5 year old South African boy is brought to the A and E deeply jaundice. He is found to have
mildly swollen, tender feet and hands. His mucosae are pale.

What is the single most likely diagnosis?

a. Di George syndrome
b. Fragile X syndrome
c. Acute myeloid leukaemia
d. Sickle cell disease
e. Edwards syndrome

167. A 6 year old Asian boy gets regular blood transfusions for his haematological abnormality. His
mucosae are pale and skull is grossly bossed. Haematological investigations were done and are as
follows:

MCH 25 g/dl
HB 8 g/dl
MCV 74 fl

What is the single most likely diagnosis?

a. Patau syndrome
b. Beta thalassaemia
c. Downs syndrome
d. Williams syndrome
e. Turners syndrome

168. A 10 year old boy is brought to the A and E with a swollen right arm. His temperature is 38.5°C
and is unable to move his arm due to severe pain. Blood cultures confirm salmonella osteomyelitis.
This is his second presentation this mouth and earlier presented with an acute onset
hepatosplenomegaly associated with severe pallor. The SHO does a sodium metabisulphite test on
the patients’ blood, which turns out to be positive.

What is the single most likely diagnosis?

a. Klinefelters syndrome
b. Sickle cell tarit
c. Patau syndrome
d. Beta thalassemia
e. Downs syndrome

169. A 39 year old male is getting progressively forgetful and is later found to have Alzheimer’s
disease. He has small ears and an IQ score of 67.
What is the single most likely diagnosis?

a. Klinefelters syndrome
b. Sickle cell tarit
c. Patau syndrome
d. Beta thalassemia
e. Downs syndrome

170. A 4 year old girl looks wasted and appears short for herage. The mother reports the daughter
has been vomiting on several occasions in the past with associated diarrhoea. The SHO thinks she
has an enteropathy and on serology, anti-endomysial antibodies were found.

What is the single most likely diagnosis?

a. Chronic non-specific diarrhoea


b. Irritable bowel syndrome
c. Celiac disease
d. Endometriosis
e. Ulcerative colitis

171. A mother brings her 5 year old son who has been passing bloody stool associated with severe
abdominal pain. On examination he is found to have mildly swollen tender wrists and red nodular
tender lesions were found on his forearms.

What is the single most likely diagnosis?

a. Endometriosis
b. Ulcerative colitis
c. Acute on chronic appendicitis
d. Giardiasis
e. Enterobius vermicularis infection

172. An 8 year old girl has got repeated episodes of diarrhoea. On each occasion the stool contains
segments of undigested vegetables. The paediatrician recommends restricting fluids to meal times.
The girl’s condition improves and she is thriving.

What is the single most likely diagnosis?

a. Chronic non-specific diarrhoea


b. Irritable bowel syndrome
c. Celiac disease
d. Endometriosis
e. Endometriosis
f. Ulcerative colitis

173. A mother and her daughter have just returned from a tropical holiday and the mother is
concerned that her daughter has an STD, since she complains of constant perianal and vulvar
irritation. There is no associated vaginal discharge.

What is the single most likely diagnosis?

a. Endometriosis
b. Ulcerative colitis
c. Acute on chronic appendicitis
d. Giardiasis
e. Enterobius vermicularis infection

174. A 15 year old girl complains of episodic diarrhoea which typically starts in the morning with a
constant urge to go to the toilet on waking and after breakfast. She says there is associated
abdominal pain in the right iliac fossa relieved by defection or flatus. She has the symptoms for 3
months.

What is the single most likely diagnosis?

a. Hirschsprungs disease
b. Acrodermatitis enteropathica
c. Intussusception
d. Chronic non-specific diarrhoea
e. Irritable bowel syndrome

175. A 5 month old baby presents with vomiting, following a 2 hr history of abdominal pain
associated with drawing up of legs. The mother says her baby has passed reddish stool.

What is the single most likely diagnosis?

a. Acute cholecystitis
b. Cyclical vomiting
c. Duodenal atresia
d. Necrotizing enterocolitis
e. Intussusception

176. A 12 year old presents with fever, flank pain. Abdominal mass is found on examination. Urine
microscopy shows no haematuria.

What is the single most likely diagnosis?

a. Wilms’ nephroblastoma
b. UTI
c. Mesenteric thrombosis
d. Acute pancreatitis
e. Gastro-esophageal reflux

177. An 8 year old girl shows signs of moderate dehydration. She has vomited all fluids for 24hrs and
the vomit is not bile stained. Her abdomen is now soft and non-tender. She has had 2 similar
episodes in the past year.

What is the single most likely diagnosis?

a. Meconium ileus
b. Acute choleystitis
c. Cyclical vomiting
d. Duodenal atresia
e. Nectrotizing enterocolitis

178. A 6 week old breastfed girl has projectile vomiting after every feed for the past 2 weeks. She is
now lethargic, dehydrated and tachypnoeic.

What is the single most likely diagnosis?


a. Hirschsprung’s disease
b. Meningitis
c. Pyloric stenosis
d. Meconium ileus
e. Acute cholecystitis

179. A 1 year old breastfed infant is vomiting after each feed. Abdominal X-ray shows a double-
bubble sign.

What is the single most likely diagnosis?

a. Cyclical vomiting
b. Duodenal atresia
c. Nectrotizing enterocolitis
d. Intussusception
e. Psychogenic vomiting

180. A 6 year old febrile child is drowsy and vomiting. She is being treated for otitis media by her GP.

What is the single most likely diagnosis?

a. Meningitis
b. Pyloric stenosis
c. Meconium ileus
d. Acute cholecytitis
e. Cyclical vomiting

181. Eight days after a premature birth, a mother notices her baby crying excessively and has passed
blood and mucus per rectum. The infant is still in the special care baby unit.

What is the single most likely diagnosis?

a. Acute cholecystitis
b. Cyclical vomiting
c. Duodenal atresia
d. Necrotizing enterocolitis
e. Intussusception

182. A 15 year old boy who is thriving has a mild abdominal pain and is passing “rice water” stools.
She had been at a mates’ birthday party the night before.

What is the single most likely diagnosis?

a. UTI
b. Mesenteric thrombosis
c. Acute pancreatitis
d. Gastroesophageal reflux
e. Gastroenteritis

183. A 15 month boy is admitted to A and E shocked. There is no history of diarrhoea, but he has
been passing large amounts of melanotic stools. Om examination he is anaemic.

What is the single most likely diagnosis?


a. Meckel’s diverticulum
b. Ulcerative colitis
c. Eosinophilic colitis
d. Crohn’s disease
e. Intussusception

184. A 4 year old girl presents with bloody diarrhoea and crampy abdominal pain. Blood tests show
anaemia and thrombocytopenia.

What is the single most likely diagnosis?

a. Haemorrhoids
b. Haemolytic- uraemic syndrome
c. Hirchsprung’s diease
d. Lymph nodular hyperplasia
e. Anal failure

185. A 5 week old infant presents with scanty streaks of fresh blood mixed with normal coloured
stools.

What is the single most likely diagnosis?

a. Meckel’s diverticulum
b. Ulcerative colitis
c. Eosinophilic colitis
d. Crohn’s disease
e. Intussusception

186. A 7 year old boy presents with streaks of fresh blood on the side of normal coloured stools and
drop of fresh blood in the toilet. There is no history of abdominal pain or rectal pain.

What is the single most likely diagnosis?

a. Haemolytic- uraemic syndrome


b. Hirchsprung’s diease
c. Lymph nodular hyperplasia
d. Anal failure
e. Juvenile polyps

187. A 3 year old boy is admitted to A and after passing several grossly bloody stools. There is no
history of abdominal pain, fever or vomiting. On examination he is markedly pain.

What is the single most likely diagnosis?

a. Meckel’s diverticulum
b. Ulcerative colitis
c. Eosinophilic colitis
d. Crohn’s disease
e. Intussusception

188. A 12 year old girl presented with a 4 week history of rectal bleedings and frequent loose
motions. She reported lower abdominal cramping during defaecation but denied fever, rash, weight
loss, arthritis or vomiting. Investigations showed anaemia but normal ESR, albumin and liver
enzymes.
What is the single most likely diagnosis?

a. Ulcerative colitis
b. Eosinophilic colitis
c. Crohn’s disease
d. Intussusception
e. Haemorrhoids

189. A 20 year old first time mother with severe weight loss, anorexia and believes her husband is
interested in killing her and their baby son and feels completely worthless.

What is the single most likely diagnosis?

a. Electrococonvulsive therapy
b. Imipramine
c. Flupenthixol
d. NO action
e. Psychodynamic psychotherapy

190. A 40 year old man has been treated for depression for 6 months. He is now being to lose weight
loss and getting ‘suicidal thoughts more frequently than before.

What is the single most likely diagnosis?

a. Electrococonvulsive therapy
b. Imipramine
c. Flupenthixol
d. NO action
e. Psychodynamic psychotherapy

191. A 29 year old woman with inability to sleep and aggressiveness and increased libido. Her
husband says prior to this, she as markedly withdrawn and blamed herself for their daughter’s death
due to cancer, threatening to join her.

What is the single most appropriate treatment option?

a. Behavioural therapy
b. Abreaction
c. Marital therapy
d. Counselling
e. Lithium

192. A 12 year old boy refuses to go to school because of constant failure to get grade. He is
threatening to starve himself to death.

What is the single most appropriate treatment option?

a. Psychodynamic psychotherapy
b. Psychosurgery
c. Cognitive surgery
d. Hypnotherapy
e. Behavioural therapy
193. A 16 year old girl with a BMI of 16 complains of 3 month history of amenorrhoea. She is not on
the pill and the pregnancy test is negative. She wants to be a model.

What is the single most appropriate treatment option?

a. Hypnotherapy
b. Behavioural therapy
c. Abreaction
d. Marital therapy
e. Counselling

194. A 20 year old man keeps cleaning his hands every time he shakes his partner’s hand. He was a
high achiever in high school.

What is the single most likely diagnosis?

a. Borderline personality disorder


b. Obsessive compulsive disorder
c. Schizotypal personality disorder
d. Avoidant personality disorder
e. Schizoid personality disorder

195. A 56 year old farmer believes that his neighbour is killing his farm animals despite the local
veterinarys advice that the death due to an outbreak of antharax. He spends hours watching his
neighbour through a pair of binocular lens, hoping to catch him in the act.

What is the single most likely diagnosis?

a. Histrionic personality disorder


b. Paranoid personality disorder
c. Borderline personality disorder
d. Obsessive compulsive disorder
e. Schizotypal personality disorder

196. A 16 year old girl is described as ‘queer’ by her mates. She is unfriendly and has no close friends
despite numerous attempts by boys in her college. She lacks empathy and is largely introspective.

What is the single most likely diagnosis?

a. Schizotypal personality disorder


b. Avoidant personality disorder
c. Schizoid personality disorder
d. Antisocial personality disorder
e. Panic attack disorder

197. A 16 year old girl in high school always comes late to lass wearing strong perfume in a vain
attempt to attract attention. She laughs loudly at jokes, many of her mates find particulary find
hilarious.

What is the single most likely diagnosis?

a. Histrionic personality disorder


b. Paranoid personality disorder
c. Borderline personality disorder
d. Obsessive compulsive disorder
e. Schizotypal personality disorder

198. A 32 year old man always gives in to his wife over any decision even when he knows that this
isn’t right. He insists on having his wife prepare his every meal, wash him and put him to sleep. He
cries whenever his wife is away.

What is the single most likely diagnosis?

a. Avoidant personality disorder


b. Schizoid personality disorder
c. Antisocial personality disorder
d. Panic attack disorder
e. Dependant personality disorder

199. A 23 year old student presents with insomnia, headaches, sweating, palpitations, chest pain and
poor appetite.

What is the single most likely diagnosis?

a. Chronic alcoholism
b. Hysteria
c. Phobia
d. Paranoid state
e. Anxiety neurosis

200. A 35 year old single woman presents with weight loss, poor appetite, decreased ability to
concentrate and guilt feelings.

What is the single most likely diagnosis?

a. Depression
b. Mania
c. Post-traumatic
d. Obsessional neurosis
e. Schizophrenia

201. A 19 year old female student presents with sudden blindness. Neurological examination reveals
no abnormality.

What is the single most likely diagnosis?

a. Obsessional neurosis
b. Schizophrenia
c. Depersonalization
d. Chronic alcoholism
e. Hysteria

202. A 20 year old man presents with disinhibition, hyperactivity, increased appetitie and grandiose
delusions.

What is the single most likely diagnosis?

a. Depression
b. Mania
c. Post-traumatic stress disorder
d. Obsessional neurosis
e. Schizophrenia

203. A 30 year old man presents with auditory hallucinations, social withdrawal and delusions of
persecution.

What is the single most likely diagnosis?

a. Schizophrenia
b. Depersonalization
c. Chronic alcoholism
d. Hysteria
e. Paranoia

204. A 21 year old man presents with compulsions and rituals, which he resists.

What is the single most likely diagnosis?

a. Post-traumatic stress disorder


b. Obsessional neurosis
c. Schizophrenia
d. Depersonalization
e. Chronic alcoholism

205. A 70 year old retired engineer experience changes in personality and impaired social skills. His
family who describe him as forgetful and not as sharp corroborates this. There are no objective
features of depression.

What is the single most likely diagnosis?

a. Dementia
b. Grief reaction
c. Panic attacks
d. Schizophrenia
e. Postnatal psychosis

206. A 20 year old man is noted to be withdrawn, isolated and ‘peculiar’. He experiences persecutory
delusions and auditory hallucinations.

What is the single most likely diagnosis?

a. Dementia
b. Grief reaction
c. Panic attacks
d. Schizophrenia
e. Postnatal psychosis

207. A 60 year old widow is noted by her family to be restless, disorganised, crying and frequently
expresses her wish to join her deceased partner.

What is the single most likely diagnosis?


a. Suicidal risk
b. Opioid abuse
c. Alcohol abuse
d. Depression
e. Anxiety

208. A 40 year old Irishman complains of frequent episodes of chest pains, sweating, palpitations, a
sense of impending doom and trembling that lasts for minutes at a time.

What is the single most likely diagnosis?

a. Anxiety
b. Delirium
c. Dementia
d. Grief reaction
e. Panic attacks

209. A 25 year old man presents with miosis, slurred speech, disorientation, and respiratory
depression.

What is the single most likely diagnosis?

a. Opioid abuse
b. Alcohol abuse
c. Depression
d. Anxiety
e. Delirium

210. A 70 year old presents with a progressive forgetfulness and mood changes. He has a shuffling
gait. The head CT scan shows cortical atrophy and enlarged ventricles.

What is the single most appropriate treatment option?

a. Propranolol
b. Donepezil
c. Dilsurfiram
d. Levodopa + carbidopa
e. Methadone

211. A 60 year old man presents with a disturbance of voluntary motor function. His face is
expressionless. On examination he has cogwest rigidity and bradykinesia

What is the single most appropriate treatment option?

a. Dilsurfiram
b. Levodopa + carbidopa
c. Methadone
d. Diazepam
e. Prozac

212. A 10 year old boy presents with brief repetitive motor ties and is brought in by his parents for
shouting obscenities at school.

What is the single most appropriate treatment option?


a. Levodopa + carbidopa
b. Methadone
c. Diazepam
d. Prozac
e. Haloperidol

213. A 40 year old man presents with ataxia. Hi wife states that it runs in her husband’s family. He is
difficult to live with, very irritable, clumsy and suffers from jerky movement of the legs.

What is the single most appropriate treatment option?

a. Long term psychotherapy


b. Tetrabenzine
c. Lithium
d. Propranolol
e. Donepezil

214. A 20 year old man presents wit sweating, muscle twitching and abdominal cramps. On
examination he had dilated pupils.

What is the single most appropriate treatment option?

a. Propranolol
b. Donepezil
c. Dilsurfiram
d. Levodopa + carbidopa
e. Methadone

215. A 20 year old female complains of irritability and constant depression. She has thought about
suicide. She has trouble sleeping and has lost her appetite.

What is the single most likely diagnosis?

a. Major depression
b. Porphyria
c. Drug-induced depression
d. Maniac- depressive disorder
e. Bereavement reaction

216. A 20 year old female complains of episodes of irritability and depression. She also complains of
monthly bloating and tension.

What is the single most likely diagnosis?

a. Premenstrual syndrome
b. Cushing’s syndrome
c. Puerperal affective disorder
d. Hyperparathyroidism
e. Major depression

217. A 30 year old complains of depression, lethargy, constipation, and weight gain. Sje also suffers
from menorrhagia
What is the single most likely diagnosis?

a. Maniac- depressive disorder


b. Bereavement reaction
c. Drug abuse
d. Hypothyroidism
e. Vitamin and mineral disorders

218. A 40 year old woman presents with depression and weight gain. She complains of back pain and
excessive thirst. Her menstrual period lasts for 3 days and sometimes she skips a cycle. On
examination she is obese with acne and peripheral edema.

What is the single most likely diagnosis?

a. Cushing’s syndrome
b. Puerperal affective disorder
c. Hyperparathyroidism
d. Major depression
e. Porphyria

219. A 30 year old female on OC pills presents with colicky abdominal pain, vomiting, and
depression. She is noted to be hypertensive. On standing, the urine turns deep red.

What is the single most likely diagnosis?

a. Porphyria
b. Drug induced depression
c. Maniac-depressive reaction
d. Bereavement reaction
e. Drug abuse

220. A 25 year old man has no close friends. He avoids social situations. He is unable to express
tender emotions. He has no overt signs of hallucination or delusional behaviour

What is the single most likely diagnosis?

a. Schizoid
b. Multiple personality
c. Borderline
d. Histrionic
e. Narcissistic

221. A 17 year old boy is brought to his GP by his parents for abusive behaviour. He is delinquent
from school and has been arrested for assault and theft.

What is the single most likely diagnosis?

a. Antisocial
b. Avoidant
c. Schizotypal
d. Passive-aggressive
e. Paranoid
222. A 23 year old man believes that he is perfect. He is unable to empathize and manipulates
people and situations. He sees everyone else as flawed. His relationship are shallow.

What is the single most likely diagnosis?

a. Borderline
b. Histrionic
c. Narcissistic
d. Antisocial
e. Avoidant

223. A 33 year old female presents to the GP with chronic feelings of boredom and depression. She
shops excessively and is sexually promiscuous. She has cut herself In the past to relive her anxiety.

What is the single most likely diagnosis?

a. Schizoid
b. Multiple personality
c. Borderline
d. Histrionic
e. Narcissistic

224. A 25 year old anxious female presents to her GP. She suffers from low level esteem doesn’t take
risks in life. She seems extremely sensitive to criticism.

What is the single most likely diagnosis?

a. Avoidant
b. Schizotypal
c. Passive-aggressive
d. Paranoid
e. Obsessive-compulsive

225. A 40 year old man insist that his wife is unfaithful and sleeping with an entire neighbourhood.
He is hypersensitive, argumentative and litigious. His wife has left him due to his behaviour. He
functions well at work.

What is the single most likely diagnosis?

a. Delusional disorder
b. Major depression
c. Paranoid schizophrenia
d. Body dysmorphic disorder
e. Postpartum depression

226. A 26 year old woman presents with personality changes. She is noted by her friends initially to
be anxious, irritable and an insomniac and weeks later she becomes profoundly depressed with low
self-esteem and contemplates suicide. In consultation she has pressured speech with boundless
energy.

What is the single most likely diagnosis?

a. Schizophrenia disorder
b. Bipolar disorder
c. Delusional disorder
d. Major depression
e. Paranoid schizophrenia

227. A 20 year old woman has a mental breakdown. She has recently broke up with her boyfriend.
She has dramatic mood swings, memory loss and incoherent speech. This lasts for a month.

What is the single most likely diagnosis?

a. Post- traumatic stress disorder


b. Brief reactive psychosis
c. Schizoaffective disorder
d. Bipolar disorder
e. Delusional disorder

228. A 20 year old woman presents to her GP complaining of feeling depressed ever since she can
remember. Her parents died in a car crash 10 years ago. She sees herself as a failure but functions
well at work. She has trouble falling asleep.

What is the single most likely diagnosis?

a. Paranoid schizophrenia
b. Body dysmorphic disorder
c. Postpartum depression
d. Panic disorder
e. Dysthymia

229. A 50 year old woman complains of sudden episodes of feeling impending doom. During these
episodes she feels choked and sweats profusely.

What is the single most likely diagnosis?

a. Paranoid schizophrenia
b. Body dysmorphic disorder
c. Postpartum depression
d. Panic disorder
e. Dysthymia

230. A 2 year old present with anaemia and abdominal pain. His mother states that she has seen him
peeling paint chips off the wall and wonders if he has been eating this?

What is the single most likely diagnosis?

a. Pica
b. Algophobia
c. Formication
d. Delirium
e. Alcohol withdrawal

231. A 23 year old woman becomes afraid to leave her home. She functions normally except will not
step outside the house.

What is the single most likely diagnosis?


a. Schizophrenia
b. Opioid withdrawal
c. Xenophobia
d. Drug toxicity
e. Agoraphobia

232. An 18 year old man presents with nausea, vomiting and diaphoresis. His pupils are dialted and
his blood pressure is elevated. He has a history of drug addiction.

What is the single most likely diagnosis?

a. Alcohol intoxication
b. Cocaine intoxication
c. Schizophrenia
d. Opioid withdrawal
e. Xenophobia

233. A 30 year old woman is found in an amnesic state. Her husband reports that she had been
missing for a few days after she has been served with divorce papers.

What is the single most likely diagnosis?

a. Alcohol withdrawal
b. Fugue
c. Alcohol intoxication
d. Cocaine intoxication
e. Schizophrenia

234. A 30 year old man with bipolar disorder is taking lithium. He was recently started on thiazide
diuretics for mild hypertension. He is now confused with ataxia, blurred vision and a coarse termor.

What is the single most likely diagnosis?

a. Cocaine intoxication
b. Schizophrenia
c. Opioid withdrawal
d. Xenophobia
e. Drug toxicity

235. A 28 year old female presents with lower abdominal pain. On examination she has multiple
surgical scars over her abdomen. Her abdominal and pelvic examinations are normal. She insists she
needs laparoscopy.

What is the single most likely diagnosis?

a. Munchausen’s syndrome
b. Dissociative disorder
c. Alcohol withdrawal delirium
d. Malingering disorder
e. Extra-pyramidal side effect

236. A 50 year old schizophrenic is started on haloperidol. A month later he is noted to be drooling
saliva and walking with a suffering gait. He also suffers from involuntary chewing movements.
What is the single most likely diagnosis?

a. Malingering disorder
b. Extra-pyramidal side effect
c. Parkinson’s disease
d. Hypothyroidism
e. Cushing’s syndrome

237. A 40 year old female complains of dry mouth, blurry vision and constipation. On examination
she had dilated pupils. She was started on amitriptyline for major depression.

What is the single most likely diagnosis?

a. Cushing’s syndrome
b. Hysterical neurosis
c. Autonomic side effect
d. Acromegaly
e. Anticholinergic side effect

328. A 45 year old man complains of headaches, excessive thirst and frequent urination. On
examination he is noted to have bad acne, coarse skin and has a goitre. He has moved his wedding
band to the 5th grader.

What is the single most likely diagnosis?

a. Hypothyroidism
b. Cushing’s syndrome
c. Hysterical neurosis
d. Autonomic side effect
e. Acromegaly

339. A 30 year old man presents to casualty with a dislocated shoulder. On examination, the
shoulder is found not to be dislocated. The patient insists it is dislocated.

What is the single most likely diagnosis?

a. Munchausen’s syndrome
b. Dissociative disorder
c. Alcohol withdrawal delirium
d. Malingering disorder
e. Extra-pyramidal side effect

340. A 28 year old woman lost her job 3 months ago. She attends her GP complaining of difficulty in
sleeping. She feels tired, has a poor appetite and has lost some weight. She feels worthless. She says
before she was fired from her job her boss used to record her phone calls and also heard him taking
ill about her to her colleagues. She thoughts of committing suicide and says that her husband would
not miss her if she were dead.

What is the single most appropriate treatment option?

a. Temazepam
b. Fluoxetine
c. Zopiclone
d. Buspirone
e. Risperidone

341. A 41 year old man has been brought to casualty after being found in the street acting in a
bizzare fashion. He was exposing his genitals to passers-by. In casualty he was angry and abusive. He
refuses to have any tests in the hospital.

What is the single most appropriate treatment option?

a. Risperidone
b. Lithium carbonate
c. Thioridazine
d. Haloperidol
e. Drug treatment not appropriate

342. A 21 year old single woman gave birth to her first child 2 days ago. Since the birth she has been
unable to sleep and is reluctant to hold her baby or to feed her. She is very tearful and cries for no
reason. She denies thoughts of harming the baby.

What is the single most appropriate treatment option?

a. Risperidone
b. Lithium carbonate
c. Thioridazine
d. Haloperidol
e. Drug treatment not appropriate

343. A 64 year old woman has a 3 year history of increasing confusion, loss of mobility and tremor.
She has recently developed frequent visual hallucinations and tends to cry out for no reason,
particularly at night. There is no evidence of an acute medical cause for her confusion. On
examination she is alert but disorientated and quite agitated. She has a coarse resting tremor,
increased tone in her limbs and normal reflexes.

What is the single most appropriate treatment option?

a. Temazepam
b. Fluoxetine
c. Zopiclone
d. Buspirone
e. Risperidone

344. A mother is worried about her 25 year old son. He seems to jump from 1 topic to another, make
up words, laugh at inappropriate things and talk to someone imaginary. He has blacked out his
windows and complains that someone else is controlling his thoughts.

What is the single most likely diagnosis?

a. Anorexia nervosa
b. Schizophrenia
c. Mania
d. Depression with psychomotor
e. Hypomania retardation
345. A 40 year old presents to his GP with headaches, insomnia and weight gain. He is slow and
sluggish when he speaks. He describes a loss of interest in everything, a feeling of worthlessness and
often cries alone.

What is the single most likely diagnosis?

a. Anorexia nervosa
b. Schizophrenia
c. Mania
d. Depression with psychomotor
e. Hypomania retardation

346. A 23 year old woman present with an episode of generalised parathesia, “flapping” limbs, loss
of urinary incontinence and abnormal posturing of the hands. These episodes are preceded by chest
tightness and difficulty swallowing. Recently she has been feeling that parts of her made of cotton
wool.

What is the single most likely diagnosis?

a. Obsessive compulsive disorder


b. Cyclothymic disorder
c. Phobia
d. Hyperventilation syndrome
e. Acute confusional state

347. A wife cannot cope up with her husband’s moods. She says he is very labile. He goes from
being full of energy and happy one day to being so pessimistic about everything the next day.

What is the single most likely diagnosis?

a. Obsessive compulsive disorder


b. Cyclothymic disorder
c. Phobia
d. Hyperventilation syndrome
e. Acute confusional state

348. A 35 year old lady has a persistent abnormality of mood continually alternating between
euphoria and depression.

What is the single most likely diagnosis?

a. Obsessional personality
b. Immoral personality
c. Sociopathic personality
d. Asthenic personality
e. Affective disorder

349. A 36 year old gentlemen who is a perfectionist to a rigid degree.

What is the single most likely diagnosis?

a. Schizoid personality
b. Suicidal personality
c. Obsessional personality
d. Immoral personality
e. Sociopathic personality

350. A 45 year old serial killer who shows no remorse for crimes he has committed.

What is the single most likely diagnosis?

a. Schizoid personality
b. Suicidal personality
c. Obsessional personality
d. Immoral personality
e. Sociopathic personality

351. A 24 year old lady whom has had a number of failed relationships and has a tendency to
dramatize limb pains.

What is the single most likely diagnosis?

a. Hysterical personality
b. Manic personality
c. Schizoid personality
d. Suicidal personality
e. Obsessional personality

352. A shy 27 year old man who is shy of advances and tends to be aloof.

What is the single most likely diagnosis?

a. Obsessional personality
b. Immoral personality
c. Suicidal personality
d. Obsessional personality
e. Immoral personality

353. A 56 year old woman says that she sees her deceased husband un and around the house.

What is the single most likely diagnosis?

a. Behavioural therapy
b. Anti- psychotic therapy
c. Bereavement counselling
d. Anti- depressive medication
e. Chlorpromazine treatment

354. A 19 year old female university student undertakes hand washing up to 30 times in one day;
this is getting in the way of her work.

What is the single most appropriate treatment option?

a. Benzodiazepine medication
b. Cognitive therapy
c. Hypnotherapy
d. Group therapy
e. Behavioural therapy
255. A 50 year old has vagrant thoughts to be alcoholic is admitted to the ward where is being unruly
and thinks the nurses are going to kill him.

What is the single most appropriate treatment option?

a. Lithium
b. Emergency care order
c. Electroconvulsive therapy
d. Benzodiazepine medication
e. Cognitive therapy

256. A 24 year old man says that aliens are talking to him and controlling his thoughts.

What is the single most appropriate treatment option?

a. Behavioural therapy
b. Anti-psychotic therapy
c. Bereavement counselling
d. Anti- depressive medication
e. Chlorpromazine treatment

257. A 40 year old woman is scared of flying.

What is the single most appropriate treatment option?

a. Benzodiazepine medication
b. Cognitive therapy
c. Hypnotherapy
d. Group therapy
e. Behavioural therapy

258. A mother brings her child to go you with chronic diarrhoea. You have strong grounds to suspect
a fractious illness because the nursing staff have found a number of packets of laxatives in her bag.

What is the single most appropriate management option?

a. Inform the police


b. Reassure
c. Inform social services
d. Imipramine
e. Refer to a dietician

259. A 25 year old lady arrives at the local accident and emergency department after ingesting an
unknown amount of an unknown drug.

What is the single most appropriate management option?

a. Refer to a dietician
b. Counselling
c. Refer to a psychiatrist
d. Contact the GP
e. Refer to general medical team
260. You get a call to visit a pateints house. The patient had been missing for last 4 days. The
patients neighbour tells you that 4 days back the patients GP had come to visit him.

What is the single most appropriate management option?

a. Refer to a dietician
b. Counselling
c. Refer to a psychiatrist
d. Contact the GP
e. Refer to general medical team

261. A 30 year old man presents with dizziness and irritability. In the A and E the patient behaves
aggressively and abuses the nursing staff. On examination you find him to be febrile. Mild neck
rigidity is present.

What is the single most appropriate management option?

a. Refer to a dietician
b. Counselling
c. Refer to a psychiatrist
d. Contact the GP
e. Refer to general medical team

262. A young lady who use to be an IV drug abuser has now completely stop taking drugs. She comes
to you saying that her boyfriend is forcing her to take those drugs again.

What is the single most appropriate management option?

a. Inform the police


b. Reassure
c. Inform social services
d. Imipramine
e. Refer to a dietician

263. A 32 year old lady is afraid to go out in the market, whenever she goes out of the house she
comes home running and gasping for breath. She immediately locks her doors and windows after
reaching home. She is otherwise normal in the house.

What is the single most appropriate management option?

a. Inform the police


b. Reassure
c. Inform social services
d. Imipramine
e. Refer to a dietician

264. A 25 year old woman finds it difficult to leave her home. She becomes very agitated in
supermarkets and describes palpitations and difficulty breathing when in crowds.

What is the single most likely diagnosis?

a. Bipolar affective disorder


b. Panic attacks
c. Anxiety neurosis
d. Post-trumatic stress disorder
e. Agoraphobia

265. A 40 year old man complains of low mood and fatigue. He has a poor appetite and has loss of
libido. He wakes at 2am and is unable to return to sleep. He feels guilty about the death if his
mother 2 years ago.

What is the single most likely diagnosis?

a. Depression
b. Social phobia
c. Bipolar affective disorder
d. Panic attacks
e. Anxiety neurosis

266. A 20 year old man is seen in dermatology outpatients with very sore hands. He has dry, cracked
skin on all his fingers. He says he has to wash his hands at least times a day and is unable to sleep of
he has not done so.

What is the single most likely diagnosis?

a. Anxiety neurosis
b. Post- traumatic stress disorder
c. Agoraphobic
d. Specific phobia
e. Obsessive compulsive disorder

267. A 20 year old woman attends A and E complaining of sudden breathlessness and anxiety. She
describes palpitations and paraesthesia of her hands, feet amd lips. ECG shows sinus tachycardia and
O2 saturation is normal.

What is the single most likely diagnosis?

a. Depression
b. Social phobia
c. Bipolar affective disorder
d. Panic attacks
e. Anxiety neurosis

268. A 35 year old train driver is on sick leave after an incident in which a child ran onto the tracks.
He is unable to go near railways and takes excessively long route to avoid them. He is troubled by
flashbacks of the accident, which frequently disturbs his sleep.

What is the single most likely diagnosis?

a. Anxiety neurosis
b. Post- traumatic stress disorder
c. Agoraphobic
d. Specific phobia
e. Obsessive compulsive disorder

269. A 35 year old man is agitated and euphoric. He claims to be helping the prime minister with
economic policy, although this is not true when checked.
What is the single most likely diagnosis?

a. Paraphrenia
b. Mania
c. Drug- induced
d. Toxic confusional state
e. Personality disorder

270. A 20 year old man complains that all his movements are begin watched. Sometimes he feels as
though his actions are being controlled by his radio. At other times he is aware of voices describing
what he is doing.

What is the single most likely diagnosis?

a. Schizophrenia
b. Korsakoff’s psychosis
c. Psychotic depression
d. Paraphrenia
e. Mania

271. A 50 year old man complains of being pursued by the police for a crime he denies committing.
He has poor concentration and impaired short-term memory. He admits to drinking large amounts
of alcohol.

What is the single most likely diagnosis?

a. Korsakoff’s psychosis
b. Psychotic depression
c. Paraphrenia
d. Mania
e. Drug- induced

272. A 65 year old woman says that she died 3 months ago and is very distressed that nobody has
buried her. When she is outdoors hears peoples say that she is evil and needs to be punished.

What is the single most likely diagnosis?

a. Korsakoff’s psychosis
b. Psychotic depression
c. Paraphrenia
d. Mania
e. Drug- induced

273. A 40 year old teetotal woman is recovering from a hysterectomy 2 days go. At night she
becomes agitated and complains of seeing animals and children walking around the wards.

What is the single most likely diagnosis?

a. Paraphrenia
b. Mania
c. Drug- induced
d. Toxic confusional state
e. Personality disorder
274. A 30 year old male sldes slowly into a severe depression over a number of months for no
particular reason. His hobbies and interests are dropped, his social contact diminishes and his mood
lower. He visits a psychiatrist for the first time because he is becoming increasingly distressed with
disturbed sleep patterns and no appetite. He feels his future is hopeless.

What is the single most appropriate treatment option?

a. Tricyclic depressants
b. MAOI
c. Reserpine
d. Carbamazepine
e. Sodium valproate

275. A 40 year old has repeated episodes of depressed mood in response to feelings rejected and a
craving for sweets and chocolates. These reactive mod changes are accompanied by hypersomnia,
lethargy and increased appetite particularly with a preference for carbohydrates.

What is the single most appropriate treatment option?

a. Tricyclic depressants
b. MAOI
c. Reserpine
d. Carbamazepine
e. Sodium valproate

276. A 55 year old woman has mania associated with cyclic and frequent depressive features.

What is the single most appropriate treatment option?

a. Tricyclic depressants
b. MAOI
c. Reserpine
d. Carbamazepine
e. Sodium valproate

277. An 18 year old previously healthy girl waiting for her college exam result. The result is arriving
that day by post and she is suffering from acute anxiety due to the importance of the results for her
future career. Her mother consults a GP.

What is the single most appropriate treatment option?

a. SSRI
b. Lorazepam
c. St John’s Wort
d. Electroconvulsive therapy
e. Lithium

278. A 30 year old female patient with suspected manic depression suffers from severe mood
swings.

What is the single most appropriate treatment option?

a. Electroconvulsive therapy
b. Lithium
c. Tricyclic depressants
d. MAOI
e. Reserpine

279. A 64 year old man with bronchoscopically confirmed squamous cell carcinoma of the right
upper lobe has a normal FEC1 of 2.5 L and normal serum biochemistry.

What is the single most appropriate investigation to assess operability?

a. Bone scan
b. Chest CT scan
c. Differential perfusion lung scan
d. Measurement of total lung capacity
e. Sputum cytology

280. A 55 year old female is admitted with a chest infection. Investigations reveal consolidation in
the right base on the chest X-ray and urinary legionella antigen is found to be positive.

What is the single most appropriate treatment?

a. Cefotaxime
b. Clarithromycin
c. Co-amoxiclav
d. Minocycline
e. Vancomycin

281. A 17 year old boy attends the accident and emergency department having been taken ill whilst
playing football. He give a history of sudden onset of left sided chest pain followed by severe and
persistent breathlessness.

What is the single investigation most likely to provide a definitive diagnosis?

a. Blood gases
b. Chest X-ray
c. Echocardiogram
d. Electrocardiogram
e. Pulmonary function tests

282. An 85 year old man who weighs 80kg is admitted as an emergency with fever and delirium. He
gives a history of dysuria and frequency. His blood pressure (BP) is 70/50 mmHg and temperature
38.6°C. His serum creatinine is 620 µmol/l and his serum area is 46 mmol/l. The microbiologist
recommends the prescription if gentamicin.

What is the single most appropriate does of gentamicin?

a. 40mg single dose


b. 40mg once daily
c. 80mg 8 hourly
d. 80mg 12 hourly
e. 160mg single dose

283. A 55 year old man presents having recently noticed a lump in his right groin. He smokes heavily,
has a persistent cough and has previously has an appendicectomy.
What is the single most likely diagnosis?

a. Epigastric hernia
b. Femoral hernia
c. Incisional hernia
d. Inguinal hernia
e. Spigelian hernia

284. A 58 year old postmenopausal woman presents with one week history of vaginal bleeding.
Digital vaginal examination is normal.

What is the single most appropriate diagnostic test?

a. Cervical smear
b. Endometrial aspiration
c. Plasma oestrogen
d. Serum FSH/LH levels
e. Transvaginal ultrasound scan

285. A 2 year old boy is brought to the A and E department by his mother. He has unable to bear
weight since a fall from a chair last night. He is tender over the mid-tibia but there is no obvious limb
deformity.

What is the single most likely radiographic appearance of the fracture?

a. Angulated fracture
b. Compound fracture
c. Epiphyseal fracture
d. No fracture likely
e. Spiral fracture

286. An 83 year old woman was admitted with a chest infection becomes confused with poor
concentration. She is restless and frightened. She is verbally abusive and has perceptual
abnormalities. There is no significant previous psychiatric history.

What is the single most likely diagnosis?

a. Acute confusional state


b. Drug- induced psychosis
c. Lewy body dementia
d. Multi-infarct dementia
e. Psychotic depression

287. A 32 year old woman was painting her bathroom when she experienced a sudden, severe head
cache, vomited and collapsed hitting her head. She has been brought to the A and E department.
She now has a Glasgow coma scale (GCS) score of 15, temperature of 37.7°C, and blood pressure of
145/85mmHg and pulse rate of 70 beats/minute. Her physical examination is normal apart from an
abrasion over her right temple.

What is the single most appropriate initial management?

a. Admit for head injury observation


b. Carboxyhaemoglobin level
c. Computed tomography (CT) brain scan
d. Lumbar puncture
e. Skull

288. A patient with dilated cardiomyopathy and permanent atrial fibrillation (AF) has a resting heart
rate of 110bpm. Twenty four hour taped recordings show even higher uncontrolled rates,
particularly associated with exercise. He is already taking 187.5mg of digoxin and has a normal
creatinine.

What is the single most beneficial treatment?

a. Addition of beta blocker


b. Addition of verapamil
c. AV node ablation and permanent pacemaker insertion
d. DC cardioversion
e. Increase digoxin to 250mg

289. A 60 year old lady presents with increasing swelling of ankles, abdominal distension and
dyspnoea. She has a past medical history of pulmonary tuberculosis as a child and a left mastectomy
and subsequent radiotherapy 5 years previously. On examination she is apyrexial, with a sinus
tachycardia of 100bpm and blood pressure of 110/60 (paradox 8mmHg). She has significant
peripheral oedema and ascites. Her JVP is elevated at 8cm above sternal angle and demonstrates a
rapidly descent.

What is the single most likely diagnosis?

a. Cardiac tamponade
b. Constrictive pericarditis
c. Intra-abdominal neoplasm
d. Severe tricuspid incompetence
e. Superior vena cava obstruction

290. What is the single most likely physiological variables to have the greatest influence toward
increasing cardiac output during the latter stages of strenuous exertion in a healthy adult?

a. Coronary vasodilation
b. Decrease in systemic vascular resistance
c. Increased alveolar ventilation
d. Increased stroke volume
e. Increased ventricular rate

291. A 15 year old girl presents with a short history of fever, malaise and a flitting polyarthritis.
Clinical examinations reveals a soft apical systolic murmur and pericardial rub. Investigations
demonstrate elevated inflammatory markers (CRP and ESR).

What is the single most likely diagnosis?

a. Acute rheumatic fever


b. Atrial myxoma
c. Kawasaki disease
d. Sabacute bacterial endocarditis
e. Systemic lupus erythematosus
292. A 45 year old man is referred to you because of breathlessness. You diagnose a pleural effusion,
which you aspirate and fine to have an eosinophilia.

What is the single most likely diagnosis?

a. Asbestos pleural effusion


b. Drug reaction
c. Fungal infection
d. Haemopneumothorax
e. Malignant pleural effusion

293. A 24 year old male has noted for the last two months that his face is swollen in the morning. He
has lost 10kg in weight over six months. He has no other complaints. On examination his external
jugular veins are dilated. His chest X-ray shows a mediastinal mass.

What is the single most likely diagnosis of his superior venacaval obstructions?

a. Adenocarcinoma of the lung


b. Hodgkin’s disease
c. Sarcoidosis
d. Seminoma
e. Tuberculosis

294. A 6 year old boy with a birth head circumference of 29cm and short palpebral fissure is found to
be mentally retarded. The boy’s mother is on acamprostate.

What is single most likely diagnosis?

a. Duchenne’s muscular dystrophy


b. Klinefelter’s syndrome
c. Celiac disease
d. Phenylketonuria
e. Fetal alcohol syndrome

295. A 70 year old man presents with history of weight loss, pain on eating and abdominal
distension. Lain abdominal X-ray films show the ‘coffee bean’ sign. Most likely diagnosis is:

a. Carcinoma of the caecum


b. Colonic polyp
c. Diverticulitis
d. Sigmoid volvulus
e. Ulcerative colitis

296. A young female presents with a 4 day history of frequency and dysuria:

a. Cystoscopy
b. Frequency/ volume chart
c. Midstream urinary sample
d. Pad test
e. Ultrasound scan of the bladder

297. A 60 year old man with presents to you with brief collapse at home. Earlier he complained of
epigastric discomfort. He is now alert with some mild back pain and tachycardia:
a. Acute salpingitis
b. Adhesive small bowel obstruction
c. Leaking aortic aneurysm
d. Mesenteric ischaemia
e. Perforated peptic ulcer

298. A young female presents with right iliac fossa pain for 4 days associated with nausea. A dipstick
urine test is normal and she also complains of an offensive vaginal discharge:

a. Appendicitis
b. Acute salpingitis
c. Urecteric colic
d. Cervical erosion
e. UTI

299. AN old chronic cigarette smoker complains of worsening right sided earache, a sore tongue amd
difficulty talking. Most likely diagnosis is:

a. Bell’s palsy
b. Myringitis bullosa
c. Ramsay hunt syndrome
d. Ramsay hunt syndrome
e. Squamous cell carcinoma of the ear canal
f. Squamous cell carcinoma of the tongue

300. A 21 year old female with a 7 weeks history of amenorrhoea with lower abdominal pain she
fainted twice. Most likely diagnosis is:

a. Threatened abortion
b. Abruption placenta
c. Ecotopic pregnancy
d. Insertio velamentosa
e. Molar pregnancy

301. A 30 year old woman in advanced labour and normal vital signs with the foetal heart rate
dropping steadily:

a. Insertion velamentosa
b. Placenta praevia
c. Molar pregnancy
d. Abruption placenta
e. Threatened abortion

302. An HIV positive man in retrovial treatment presents with a painless lymphadenopathy fevers,
drenching night sweats and weight loss:

a. Acute myeloid leukaemia


b. Acute promyelocytic leukaemia
c. Chronic myeloid leukaemia
d. Hodgkin’s lymphoma
e. Non- Hodgkin’s lymphoma
303. An old man undergoes Hartmann’s procedure on the 8th postoperative day he develops a high
spiking fever. Most likely cause is:

a. Basal lung collapse


b. CVP line infection
c. Subphrenic abscess
d. Urinary tract infection
e. Wound infection

304. A 14 year old girl presents with a long history of up to five painful oral ulcers occurring at
intervals of 1-2 months. She is otherwise well, most likely diagnosis is:

a. Erythema multiforme
b. Oral mucositis
c. Recurrent oral ulceration
d. Shingles
e. Squamous cell carcinoma

305. A 62 year old obese, diabetic and hypertensive woman presents with two episodes of
postmenopausal bleeding. Investigations of first choice will be:

a. CT abdomen and pelvis


b. Cytology
c. Hysteroscopy and D and C
d. Diagnostic laparoscopy
e. Examination under anaesthesia and representing

306. A thin 17 year old girl with thickened calluses on the dorsum of her hand and has bilateral
parotid swelling. Most likely cause is:

a. Bulima nervosa
b. Crohn’s disease
c. Mumps
d. Sarcoidosis
e. Sjogren syndrome

307. A 28 year old obese female presents in eary labour. On vaginal examination there is an ‘empty’
pelvis. Most appropriate step of management is:

a. CTG monitoring
b. Fetal blood sampling
c. Oxygen supply
d. Ultrasound scan
e. Pregnancy test

308. CTG is showing variable decelerations in a multiparous woman in advanced labour. The next
appropriate step will be:

a. Foetal blood sampling


b. Emergency caesarean section
c. Instrumental vaginal delivery
d. Oxygen supply
e. Ultrasound scan
309. An 8 month old girl passing large offensive stools since 3 months of age. She feeds well but her
weight has gradually fallen across the centiles. She has had persistent ‘chest infections’ needing
frequent antibiotics:

a. Cystic fibrosis
b. Gluten enteropathy
c. Hirschsprung disease
d. Milk protein intolerance
e. Ulcerative colitis or chronic inflammatory bowel disease

310. An asymptomatic 18 year old girl with normal secondary characteristics and normal endocrine
results has a negative progesterone challenge test:

a. Absent uterus
b. Constitutional
c. Prolactinoma
d. Sheehan’s syndrome
e. XO-Karyotype

311. A 62 year old female presents with lethargy, thinning hair and loss of the outer third of her
eyebrows:

a. Androgenetic alopecia
b. Hypothyroidism
c. Iron deficiency
d. Malnutrition
e. Telogen effluvium

312. A 33 year old multiparous woman is admitted with a blood pressure of 160/96 mmHg and 1+
protein excretion. First investigation you would do will be. First investigation you would do will be:

a. 24 hours urine protein excretion


b. Doppler ultrasound of the umbilical artery
c. Glucose tolerance test
d. Keilhauer test
e. Ultrasound whole abdomen

313. The triple test of a 23 year old female showed that her baby was in increased risk of developing
Down’s syndrome. Your next step to confirm will be:

a. Amniocentesis
b. Doppler ultrasound of the of the umbilical artery
c. Keilhauer test
d. Ultrasound whole abdomen
e. Bile acids

314. A 52 year old obese woman complains of a burning retrosternal disco fort on lying down and on
eating.

a. Achalasia
b. carcinoma of the oesophagus
c. chronic benign stricture
d. Myasthermia gravis
e. Plummer- Vinson’s syndrome

315. A 38 year old female who is pale complains that food it sticking in the back of her throat. She is
found to have spoon- shaped nails, a smooth tongue and angular cheilitis:

a. Achalasia
b. carcinoma of the oesophagus
c. Myasthermia gravis
d. Plummer- Vinson’s syndrome
e. Reflux oesophagitis

316. A young man presents with groups of small intensely itchy vesicles affecting the skin on his
knees, buttocks, elbows and scalp:

a. Dermatitis herpetiformis
b. Eczema
c. Lichen planus
d. Liver disease
e. Scabies

317. A 20 year old boy presents to clinic with painless cervical lymphadenopathy which he noticed
while taking bath. He has been feeling lethargic for some time with night sweats and itchy skin:

a. Lymphoma
b. Liver disease
c. Tuberculosis
d. Dermatitis herpetiformis
e. Polycythaemia

318. A 48 year old female has developed increase in abdominal girth alongside with constipation and
weight loss:

a. Irritable bowel syndrome


b. Ovarian cancer
c. Pelvic endometriosis
d. Ruptured ovarian cyst
e. Torsion of an ovarian cyst

319. A patient complains of difficulty opening the eyes of waking with a purulent discharge and
discomfort from both eyes. He has inflamed conjunctivae. There is no less of visual acuity:

a. Acute angle closure glaucoma


b. Bacterial conjunctivitis
c. Chlamydial conjunctivitis
d. Scleritis
e. Viral conjunctivitis

320. A 32 year old dentist complains of a painful red eye and photophobia with watery discharge.
Dendritic corneal ulceration are seen with fluorescein drops:

a. Ulcerative keratitis
b. Aute angle closure glaucoma
c. Viral conjunctivitis
d. Bacterial conjunctivitis
e. Anterior uveitis

321. Following the removal of his plaster for a fracture of the distal radius and ulna after 2 weeks a
15 year old boy notes a deformity of his right wrist. Which one of the following is the most likely
cause?

a. Compartment syndrome
b. Growth plate disturbance/damage
c. Malunion
d. Muscle haematoma
e. Tendon rupture

322. Erb’s point is formed by the confluence of BDC 26:

a. C7, C
b. C5, C6
c. C8, C6
d. C2, C3
e. C4, C5

323. Warfarin- induced skin neurosis is due to:

a. Protein C deficiency
b. Protein S deficiency
c. Antithrombin III defect
d. Increased factor VII
e. Increased platelet count

324. True about supraglottic carcionoma:

a. Cigarette smoking is the most important predisposing


b. Rarely presents with neck nodes
c. Hot potato voice is a feature
d. Treatment is surgery with adjuvant chemotherapy
e. More common in females

325. All the following are true about ludwigs angina except:

a. It is an infection of the floor of the mouth


b. It may involve the sublingual and submandibular spaces
c. It is caused by H. influenzae
d. Treatment is with antibiotics
e. Caused by anaerobic organisms

326. Cephalic index is useful for determination of:

a. Age
b. Sex
c. Race
d. Religion
e. Caste

327. Prolongation of rigor mortis is seen in:


a. Lead
b. Arsenic
c. Mercury
d. Copper

328. Which of the following conditions is asscoaited with secreased levels of serum alpha-
fetoprotein in the mother:

a. Pre- eclampsia
b. IUGR
c. Trisomy 21
d. Multiple pregnancy
e. Large baby

329. A 26 year old woman with a dead foetus in transverse lie and with hand prolapse is now in
second stage of labour. The management would include:

a. Craniotomy
b. Decapitation
c. Cleidotomy
d. Caesarean section
e. Amniotomy

330. In bow presentation, the engaging diameter is:

a. Submentrovertical
b. Mentrovertical
c. Submentobregmatic
d. Suboccipitofrontal
e. Suboccipitobregmatic

331. Signs of heart disease in pregnancy:

a. Diastolic murmur
b. Systolic murmur
c. Tachycardia
d. Dyspnoea or exertion
e. Nervousness or syncope on exertion

332. All the following cells are in seen in Hodgkin’s disease except:

a. Hodgkin cells
b. L and H cells
c. Langerhans’ cells
d. Lacunar cells
e. Reed- sternburg cells

333. When hypoglycemic coma is suspected, then glucose is to be infused based on:

a. Clinical assessment only


b. After consulting endocrinologist
c. After blood glucose estimation
d. After seeing for urine glucose
e. After ruling out electrolyte imbalance

334. Causes of palpable purpura include:

a. HUS
b. DIC
c. Henoch-schonlein purpura
d. TTP

335. The schilling test:

a. Requires marrow to be aspirated from the sternum


b. Involves labelling with radioactive chromium
c. Can determine the rate at which red cells are destroyed D may confirm a diagnosis of
pernicious anaemia
d. Sometimes indicates the need for splenectomy

336. In a patient with lower ulnar nerve palsy which of the following would be seen:

a. Loss of extension at carpometacarpal


b. Sensory loss over dorsal surface of medial 4 fingers
c. Loss of abduction at thumb
d. Weakness of hand movements
e. Thenar wasting

337. All the following are autosomal recessive except:

a. Phenylketonuria
b. Alpha 1 antitrypsin deficiency
c. Hereditary spheracytosis
d. Gauchers disease
e. Hereditary hemochromatosis

338. Causative agent for pseudomembranous colitis include:

a. Staphylococci
b. Clostridium difficile
c. Streptococcus
d. Pseudomonas
e. Candida albicans

339. Adventitial bursitis due to tuberculosis is found in which of the following sites commonly:

a. Prepateeler
b. Greater trochanter of femur
c. Subacromial
d. Metatarsal
e. Subolecranon

340. True about etiopathogeneis of Alzheimer’s disease include:

a. APP gene defect


b. Prion disease
c. Autoantibody formation
d. Herpes infection

341. Call exner bodies are seen in:

a. Leydigs cell tumour


b. Granulosa cell tumour
c. Embryonal cell carcinoma
d. Endodermal sinus tumour
e. Brenner’s tumour

342. True regarding psychological jaundice includes:

a. It occurs due to increased RBC destruction


b. Indication to stop breastfeeding
c. Lasts up to 3 weeks
d. Appears before 24 hours
e. Due to biliary obstruction

343. What is the age of the child when he can follow horizontally 180° has got almost head control
and cannot sit without support:

a. 2 months
b. 3 months
c. 6 months
d. 9 months
e. 10 months

344. Infant with Down’s syndrome have all except:

a. VSD
b. Duodenal atresia
c. Leukaemia
d. Normal intelligence
e. Delayed skeletal maturation

345. Antagonists to opium poisoning’s include:

a. Neostigmine
b. Nalophine
c. Buprenorphine
d. Pyridostigimine
e. Meptazinol

346. In a normal person at resting condition GFR in mL/ min is:

a. 125
b. 90
c. 60
d. 150
e. 2000

347. Flight of ideas is seen which of the following diseases:

a. Depression
b. Schizophrenia
c. Mania
d. Obsessive disorders
e. Hysteria

348. A homosexual personal feel that he is imposed by a female body and persistenet discomfot
with his sex. The diagnosis is:

a. Gender identity disorder


b. Transvestism
c. Voyeurism
d. Paraphilia

349. In a patient with intestinal obstruction which of the following investigations would be required:

a. X-ray abdomen
b. Barium meal
c. Upper GI endoscopy
d. Ultrasonography
e. CT abdomen

350. Henoch- schonlein purpura may be distinguished from post- streptococcal nephritis by the
following finding:

a. Swollen joints
b. High temperature
c. Red cells casts in the urine
d. Arthralgia
e. Melaena
1-300
Question Answer Question Answer Question Answer Question. Answer Question Answer
NO. NO. NO. NO NO.
1 B 61 E 121 D 181 D 241 D
2 C 62 C 122 A 182 E 242 E
3 B 63 E 123 A 183 A 243 E
4 A 64 A 124 B 184 B 244 B
5 B 65 A 125 B 185 C 245 D
6 C 66 B 126 D 186 E 246 D
7 E 67 A 127 B 187 A 247 B
8 D 68 C 128 A 187 A 248 E
9 A 69 E 129 D 188 B 249 C
10 C 70 B 130 A 190 A 250 E
11 C 71 E 131 A 191 E 251 B
12 A 72 A 132 A 192 C 252 B
13 A 73 C 133 E 193 B 253 C
14 E 74 B 134 E 194 B 254 E
15 E 75 D 135 B 195 B 255 D
16 E 76 B 136 E 196 C 256 B
17 D 77 A 137 A 197 A 257 E
18 D 78 A 138 E 198 E 258 C
19 A 79 D 139 A 198 E 259 E
20 A 80 A 140 C 199 A 260 D
21 B 81 C 141 A 200 E 261 E
22 D 82 E 142 E 201 B 262 A
23 A 83 A 143 A 203 A 263 B
24 A 84 A 144 B 204 B 264 E
25 D 85 B 145 C 205 A 265 A
26 E 86 E 146 D 206 D 266 E
27 D 87 A 147 E 207 A 267 E
28 E 88 B 148 A 208 E 268 B
29 B 89 D 149 A 209 A 269 B
30 B 90 E 150 C 210 B 270 A
31 A 91 B 151 D 211 B 271 A
32 A 92 A 152 B 212 E 272 B
33 D 93 C 153 A 213 B 273 C
34 A 94 A 154 D 214 E 274 A
35 D 95 B 155 D 215 A 275 B
36 D 96 E 156 C 216 A 276 E
37 A 97 A 157 A 217 D 277 B
38 B 98 D 158 A 218 A 278 B
39 E 99 D 159 D 219 A 279 B
40 A 100 C 160 A 220 A 280 B
41 B 101 E 161 A 221 A 281 B
42 E 102 E 162 C 222 C 282 E
43 E 103 A 163 B 223 C 283 D
44 A 104 D 164 C 224 A 284 E
45 A 105 C 165 A 225 A 285 E
46 E 106 A 166 D 226 B 286 A
47 A 107 A 167 B 227 B 287 C
48 A 108 D 168 D 228 E 288 E
49 A 109 C 169 E 229 D 289 B
50 C 110 D 170 C 230 A 290 E
51 C 111 C 171 B 231 E 291 A
52 B 112 D 172 A 232 B 292 E
53 E 113 A 173 E 233 B 293 B
54 A 114 C 174 E 234 E 294 E
55 A 115 D 175 E 235 A 295 D
56 C 116 A 176 A 236 B 296 C
57 D 117 A 177 C 237 E 297 C
58 E 118 A 178 C 238 E 298 A
59 D 119 A 179 B 239 D 299 E
60 C 120 C 180 A 240 B 300 C
301-350
Question NO. Answer Question NO. Answer
301 A 326 C
302 E 327 BC
303 C 328 C
304 C 329 B
305 C 330 B
306 A 331 A
307 D 332 C
308 A 333 A
309 A 334 C
310 A 335 D
311 B 336 D
312 A 337 C
313 A 338 B
314 E 339 B
315 A 340 A
316 A 341 B
317 A 342 B
318 B 343 C
319 B 344 D
320 A 345 B
321 C 346 A
322 B 347 C
323 A 348 A
324 A 349 A
325 A 350 E

You might also like